Pastest-Basic Sciences PDF
Pastest-Basic Sciences PDF
[ Q: 1269 ] PasTest 2009 - Basic 1- Kappa light chains in the urine (Bence Jones
Science protein)
A patient presents with eczema, 2- Normal IgG and IgM
thrombocytopenia and recurrent infections. 3- Reduced plasma viscosity
What is the most likely diagnosis? 4- Elevated b 2-glycoprotein-1
1- Wiskott-Aldrich syndrome 5- Elevated serum creatinine
2- Hyper-IgE syndrome
3- Gaucher's disease Answer & Comments
Which of the following tests most supports a Answer: 3- Antimitochondrial antibodies (M2
malignant process? pattern)
M2 AMA is highly indicative of PBC. A fourth branchial arches, and therefore the
polyclonal increase in IgM (sometimes thymus and parathyroid, fail to develop. As a
associated with elevated IgG) is typical but not result, cardiac outflow tract anomalies occur
specific for PBC. Raised serum IgA and anti- (interrupted aortic arch, truncus arteriosus
smooth muscle antibodies would not support and teratology of Fallot).
a diagnosis of PBC.
[ Q: 1273 ] PasTest 2009 - Basic
[ Q: 1272 ] PasTest 2009 - Basic Science
Science
A 22-year-old woman with partial
Which of the following diseases is correctly lipodystrophy presents with a 3- month
matched to the immunodeficiency? history of increasing swelling of her legs,
1- Ataxia-telangiectasia - absent NBT which is now up to her knees. Urinalysis shows
(neutrophil nitroblue tetrazolium) heavy proteinuria but no haematuria. A
reduction diagnosis of nephrotic syndrome is therefore
made.
2- Bruton's disease - impaired phagocytosis
Which investigation is most likely to help in the
3- Chronic granulomatous disease (CGD) - definition of the underlying renal pathology?
hypogammaglobulinaemia
1- Serum immunoglobulins
4- Chèdiak-Higashi - reduced IgA levels
2- Complement studies
5- DiGeorge syndrome - absent T-cell function
3- Antineutrophil cytoplasmic antibodies
(ANCA)
Answer & Comments
4- Antiglomerular basement membrane
Answer: 5- DiGeorge syndrome - absent T-cell antibodies (anti-GBM)
function
5- Antistreptolysin titre (ASOT)
Ataxia-telangiectasia presents in childhood
with cerebellar ataxia, impaired cell-mediated Answer & Comments
immunity and productions of antibody. It is an
autosomal-recessive condition. Bruton's Answer: 2- Complement studies
disease is an X-linked Nephrotic syndrome in an individual with
hypogammaglobulinaemia due to absent partial lipodystrophy is likely to be
mature B cells. Recurrent pyogenic infections mesangiocapillary nephritis with C3 nephritic
occur once maternal antibody levels fall. CGD factor. The latter is an autoantibody, and
(chronic granulomatous disease) is caused by interacts with the complement pathway to
a failure of intracellular killing (no respiratory give a low C3 with a normal C4 level. Serum
burst). There are various types. Screening is by immunoglobulins are likely to show reduced
the nitroblue tetrazolium (NBT) test. IgG and raised IgM levels, which is typical of
Ch©أdiak-Higashi is a disorder affecting the nephrotic syndrome. Anti-GBM antibodies
neutrophil chemotaxis and is inherited as an are likely to be negative as the patient is
autosomal-recessive. DiGeorge syndrome young and there is no haematuria or
consists of hypoparathyroidism pulmonary involvement.
(hypocalcaemic convulsions in newborns),
cardiac anomalies, abnormal facies and absent
T-cell function (impaired cell-mediated
immunity). In this condition the third and
A medical SHO is required to give a blood The major histocompatibility complex (MHC)
sample to check his HepB status. He received is a cluster of genes located on the short arm
a course of vaccinations nine months ago. of chromosome 6, and it codes for a series of
What is his blood test likely to show? molecules known as the ‘human leucocyte
antigens' (HLA). HLA molecules are distributed
1- Anti-HBeAb throughout the body, and it is through
2- Anti-HBsAb differential HLA expression that cells are
classified as ‘self' or ‘non-self'. Class-I
3- Anti-HBsAb + anti-HBcAb
molecules (subtypes A, B and C) are expressed
4- HBsAg + HBcAg on all cell types except erythrocytes and
5- IgM to HBcAg trophoblasts. They interact with CD8-positive
T cells and are involved in driving cytotoxic
Answer & Comments reactions.
Low-titre antinuclear antibodies are often a [ Q: 1292 ] PasTest 2009 - Basic Science
normal finding in middle-aged and older
Susceptiblity to Pneumocystis jiroveci can be
women.
associated with high levels of:
1- IgM
[ Q: 1291 ] PasTest 2009 - Basic
Science 2- IgG
2- Cytotoxic T cells are activated before 4- The normal colloid oncotic pressure is 70
natural killer cells during the course of mmHg
infection 5- Albumin is indicated
3- Viruses stimulate the non-immune cells
that they infect to produce interferon-g Answer & Comments
4- Non-enveloped viruses are susceptible to
Answer: 1- Certain intravenous solutions,
damage by complement
which would be hypo-osmolar, have dextrose
5- Influenza virus can avoid antibody added to ensure they are iso-osmolar
recognition by mutational changes in its
nucleocapsid proteins Hypovolaemia is an important factor
contributing to shock and tissue hypoxia;
therefore, all patients with sepsis require
Answer & Comments supplemental fluids. The amount and rate of
Answer: 1- IgA can offer protection at mucosal infusion are guided by an assessment of the
surfaces patient's volume and cardiovascular status.
Patients withs suspected septic shock require
Natural killer cells are activated faster than an initial crystalloid fluid challenge of 20-30
cytotoxic T cells. Infected non-immune cells ml/Kg (1-2 l) over a period of 30-60 minutes.
produce interferon-a and - b, whereas Fluid resuscitation is continued until heart
interferon-g is produced by T cells. Influenza rate, urine output, and blood pressure are
virus mutates its surface neuraminidase and satisfactory and/or the pulmonary capillary
haemagglutinin to avoid antibody recognition. wedge pressure exceeds 18 mmHg. Patients
Enveloped viruses are susceptible to with septic shock often require a total of 4-6 l
complement attack. or more of crystalloid resuscitation; this need
for relatively large volumes to restore
[ Q: 1304 ] PasTest 2009 - Basic intravascular volume is a disadvantage of
Science crystalloid solutions. Colloids on the other
hand are much more expensive and colloid
A 76-year-old patient is admitted to the acute
resuscitation (with albumin or hetastarch) has
admission unit with septic shock. Pulse is 106
not previously been shown in meta-analyses
and BP 90/40 mmHg. Urinary catheterisation
to have any benefit over isotonic crystalloid
produces 75 ml of concentrated urine.
resuscitation (isotonic sodium chloride
Which of the following principles applies to the solution or lactated Ringer solution).
choice of an appropriate intravenous fluid for Hartmann's solution contains sodium,
resuscitation? potassium, chloride, calcium and lactate. It is
1- Certain intravenous solutions, which would not possible to store bicarbonate in solution
be hypo-osmolar, have dextrose added to with calcium; instead lactate is used which is
ensure they are iso-osmolar metabolised to bicarbonate by the liver.
2- Hartmann's solution contains sodium,
potassium, chloride, calcium and [ Q: 1305 ] PasTest 2009 - Basic
bicarbonate Science
3- An advantage of crystalloid solutions is that Which of the following takes place during
relatively small volumes have to be infused inspiration?
to restore an intravascular volume deficit 1- The diaphragm drops by 10 cm during
normal breathing
stage. A large pulsatile increase in GnRH 2- Low-frequency waves are detected in the
occurs just before ovulation. scala vestibuli
3- The scala media is filled with perilymph
[ Q: 1308 ] PasTest 2009 - Basic
4- The scala media contains the organ of Corti
Science
5- Normal hearing frequency only ranges from
A 24-year-old woman visits her GP 20 to 2000 Hz
complaining that she is tired and lethargic. He
arranges some routine blood tests including
Answer & Comments
FBC, U&E, LFT, TFT, viscosity and
immunoglobulins. The only abnormality is Answer: 4- The scala media contains the organ
elevated IgE levels to 1.2 times the upper limit of Corti
of normal.
Low-frequency waves are detected in the
Raised IgE levels are a normal finding in what scala tympani, high-frequency waves are
% of the population? detected in the scala vestibuli. Both contain
1- 2% perilymph. The scala media is filled with
potassium-rich endolymph. Normal hearing
2- 5%
frequency ranges from 20 to 20,000 Hz.
3- 2.5%
4- 1.25% [ Q: 1310 ] PasTest 2009 - Basic
5- 10% Science
Whilst treating a diabetic patient who is
Answer & Comments controlled with insulin you perform
fundoscopy and find new vessel disease in the
Answer: 3- 2.5%
macula lutea.
Reference ranges for laboratory variables are Which of the following is true of ocular
calculated based on sample values from a physiology?
series of normal volunteers. IgE levels are
normally distributed, and abnormal values are 1- The macula is rich in rods
said to lie outside 2 standard deviations either 2- This corresponds with the optic papilla
side of the mean. In a normal distribution 95% 3- The central portion of the macula is known
of values lie within 2 standard deviations as the fovea centralis
either side of the mean. As such 2.5% are
lower than 2 standard deviations less than the 4- As it corresponds with the blind spot,
mean, and 2.5% are greater than 2 standard retinopathy will have less functional effect
deviations above the mean. on acuity
5- There is a direct relationship between
[ Q: 1309 ] PasTest 2009 - Basic proliferative retinopathy and the frequency
Science of cataracts
corresponds with the blind spot. The fovea concentration of carbon dioxide in the blood
centralis has the highest acuity. There is no rises, a condition known as ‘hypercapnia', the
direct relationship between proliferative central and peripheral chemoreceptors
retinopathy and the frequency of cataracts. stimulate the inspiratory areas to stimulate
the rate and depth of breathing. This
[ Q: 1311 ] PasTest 2009 - Basic hyperventilation causes more carbon dioxide
Science to be exhaled until levels return to normal.
3- Central chemoreceptors are sensitive to the 2- The cilia are under the control of a
H+ content of the CSF physiological motor, dynein
4- The carotid bodies are located on the 3- The mucociliary escalator moves at 0.2
external carotid artery cm/minute
5- Central chemoreceptors are sensitive to the 4- The bronchioles have cartilage in their wall
O2 content of the CSF 5- The bronchioles have diameters up to 5 mm
The chemoreceptors that regulate respiration About 100 ml of mucus is produced every day.
are located both centrally and peripherally. The cilia are under the control of a
Normally control is exercised by the central physiological motor, dynein (which is absent
receptors located in the medulla, which in Kartagener's syndrome). The mucociliary
respond to the CSF hydrogen ion escalator moves at 2 cm/minute. The
concentration, in turn determined by CO2, bronchioles do not have cartilage in their wall
which diffuses freely across the blood-brain (which distinguishes them from bronchi). The
barrier from the arterial blood. The response bronchioles can be up to 1 mm in diameter.
is both quick and sensitive to small changes in
arterial CO2 (PaCO2). Whilst central [ Q: 1313 ] PasTest 2009 - Basic
chemoreceptors are therefore (indirectly) Science
sensitive to levels of carbon dioxide,
peripheral chemoreceptors are also highly In estimating the physiological clearance of 10
sensitive to oxygen. There are three clusters ml of an intravenous substance which has
of peripheral chemoreceptors: the aortic body been administered at 10 mg/ml, the plasma
located in the wall of the arch of the aorta, concentration at equilibration is 15 mg/litre,
and the two nodular carotid bodies in the left the urine concentration is 150 mg/litre and
and right common carotid arteries. When the the subject produces 1440 ml of urine during a
24h collection. The substance is not actively 4- Only about 0.15% of oxygen is carried in
secreted or absorbed by the kidney. solution in the plasma
What is the clearance of the substance? 5- Carbon dioxide is less water-soluble than
1- 1 ml/min oxygen
2- 10 ml/min
Answer & Comments
3- 0.1 ml/min
Answer: 1- Gas exchange can occur in the final
4- 100 ml/min seven branches of the bronchoalveolar tree
5- Cannot say from the information given
Gas exchange can occur in the final seven
branches of the bronchoalveolar tree (the
Answer & Comments respiratory zone). The first 16 branches of the
Answer: 2- 10 ml/min bronchial tree are collectively known as the
conducting zone. The equilibration of gases
Clearance is calculated using the formula (U —أ takes about 0.25 s in the resting lung. Only
V)/P where U = urine concentration in mg/ml, about 1.5% of oxygen is carried in solution in
V = urine production in ml/min, P = plasma the plasma. Carbon dioxide is more water-
concentration in mg/ml. soluble than oxygen, between 5 and 10% of
Using the values here, this gives: CO2 is carried in dissolved form.
[ Q: 1316 ] PasTest 2009 - Basic 2- Antibodies to ABO blood groups are IgG,
Science whereas antibodies to Rhesus antigens are
IgM
Which of the following is true
concerning complement activation? 3- Antibodies to ABO blood groups are IgA,
whereas antibodies to Rhesus antigens are
1- IgG and IgE are the main antibody classes
IgG
involved in classical pathway activation
4- Antibodies to Rhesus antigens are IgD,
2- C1q binds to the Fab regions of antigen-
whereas anti-ABO blood groups are IgM
complexed IgG antibodies
5- Antibodies to Rhesus antigens are IgE,
3- The alternative, but not the classical C3,
whereas anti-ABO blood groups are IgG
convertase enzyme involves C3b
4- Elevated serum C3dg is a good marker of Answer & Comments
complement activation
Answer: 1- Antibodies to ABO blood groups
5- The membrane-attack complex involves
are IgM, whereas antibodies to Rhesus
polymerisation of C7
antigens are IgG
interact with CD4 T cells. CD8 T cells interact [ Q: 1327 ] PasTest 2009 - Basic Science
via MHC class I and this MHC class can be
Where is the antibody targeted in Lambert-
expressed on any cell in the body. T cell
Eaton syndrome?
interactions are regulated by expression of
different MHC antigens, resulting in the 1- Anticholinesterase
association between a number of 2- Mitochondria
autoimmune diseases and specific MHC
3- Sodium channels
subtypes.
4- Potassium channels
[ Q: 1326 ] PasTest 2009 - Basic 5- Voltage-gated calcium channels
Science
Where is the immune defect in chronic Answer & Comments
lymphocytic leukaemia situated? Answer: 5- Voltage-gated calcium channels
1- Complement
Lambert-Eaton myasthenic syndrome (LEMS)
2- Immunoglobulin G (IgG) is a rare condition where muscle weakness is
3- Macrophages the result of abnormal acetylcholine (ACh)
release at the neuromuscular junction. It was
4- Mast cells
first described by Lee McKendre Eaton, an
5- B cells American neurologist, with Edward H.
Lambert and E. D. Rooke. Antibodies against
Answer & Comments presynaptic voltage-gated calcium channels
(VGCC) are believed to play an important role
Answer: 5- B cells in the pathogenesis by inhibiting the calcium
Chronic lymphocytic leukemia (CLL) is a currents that trigger ACh release. A serum test
malignancy of B lymphocytes. In about 50% of for voltage-gated calcium channel antibodies
cases, a diagnosis of CLL is incidental (to some is now available.
other condition); it requires a white blood cell
count and peripheral smear, with definitive [ Q: 1328 ] PasTest 2009 - Basic
diagnosis by flow cytometric Science
immunophenotype.
A dental practice nurse is admitted for an
The median survival after diagnosis is about 7 elective operation. During induction with a
years. One interesting aspect of CLL is that it general anaesthetic she develops tachycardia,
results in defective antigen presentation with rash and diffuse wheeze. She had mentioned
an acquired T cell dysfunction, and manifests that she has had rashes when assisting with
as both an alloimmune and autoimmune anaesthetics at work.
defect. Abnormal antibodies are produced in What is the likely diagnosis?
about 30% of patients, and they are produced
not by the malignant cells but by healthy 1- Systemic mastocytosis
polyclonal cells. These antibodies are typically 2- Anaphylaxis
target restricted for blood cell antigens. 3- Pseudoallergy
4- Serum sickness
5- Contact dermatitis
5- Chromosomal non-dysjunction
[ Q: 1340 ] PasTest 2009 - Basic
Science
Answer & Comments
A couple aged in their early 30s come to the
Answer: 4- Autosomal recessive genetics clinic. They have one 6-year-old son
This boy has Wolfram syndrome, a rare who has gross obesity and mild mental
progressive neurodegenerative disorder impairment, diagnosed as Prader Willi
characterised by diabetes insipidus, diabetes syndrome and they are concerned about the
mellitus, optic atrophy and sensorineural possibility of further children being affected.
deafness (DIDMOAD). Patients may also suffer Which of the following most accurately
from red-green colour blindness. describes the mode of inheritance of Prader
Heterozygotes are thought to have an Willi?
increased risk of Type 1 diabetes. Carrier
1- Autosomal recessive
frequency is around 1:350, leading to a
prevalence of 1:770,000. A variety of 2- Autosomal dominant
mutations in the WFS1 gene are thought to be 3- X-linked recessive
responsible.
4- X-linked dominant
5- Non-mendelian
[ Q: 1339 ] PasTest 2009 - Basic
Science
Answer & Comments
Which of the following disorders may have an
autosomal recessive mode of inheritance? Answer: 5- Non-mendelian
1- Achondroplasia Prader Willi occurs because of deletion or
2- Glucose 6-phosphate dehydrogenase disruption of genes on the proximal arm of
deficiency chromosome 15, (15q11-13). It is the first
human disorder to be attributed to genetic
3- Huntington's disease imprinting, where genes are expressed
4- Ehlers-Danlos syndrome Type IV differently based on which parent they came
5- Haemophilia A from. Around 70% of cases occur due to
deletion of 15q11-13, 28% due to maternal
disomy. Prevalence of the condition is
Answer & Comments
estimated as being as high as 1:8000 in rural
Answer: 4- Ehlers-Danlos syndrome Type IV Sweden, and as low as 1:25000 in other areas.
Features include morbid obesity, hyperphagia,
Ehlers-Danlos syndrome is a autosomal hypogonadism and mental impairment.
recessive heritable disorder of connective
tissue with easy bruising, joint hypermobility
[ Q: 1341 ] PasTest 2009 - Basic
(loose joints), skin laxity, and weakness of
Science
tissues. There are a number of different types
of Ehlers-Danlos syndrome (EDS) which share In which of the following genetic diseases is
the foregoing features but can be categorised DNA analysis useful?
into at least nine different types. Type IV may
1- Adult polycystic kidney disease
2- Down's syndrome
one of the most common single gene [ Q: 1337 ] PasTest 2009 - Basic
mutation disorders and has a prevalence said Science
to be between 1 in 3000 and 1 in 10,000. It
Which of the following should be
carries an autosomal dominant inheritance
considered in the management of haemophilia
pattern with a relatively high clinical
A?
variability. Two prominent systems of clinical
classification, (Berlin and Ghent) are used to 1- The infant is protected at birth due to
characterise the disorder. maternal transfer of factor VIII
2- Desmopressin may be useful
[ Q: 1336 ] PasTest 2009 - Basic 3- A factor VIII concentration < 10% causes
Science severe disease
A 19-year-old man from a family of travellers 4- Most cases are the result of new mutations
presents to the Emergency department. He
5- von Willebrand factor levels are reduced
has suffered a sudden deterioration in vision.
Additional past medical history of note
includes bilateral shoulder dislocation. On Answer & Comments
examination he is tall and thin with a high Answer: 2- Desmopressin may be useful
arched palate. He appears to have suffered a
lens dislocation Because factor VIII is not transferred through
the placenta, bleeding may occur in severe
Which of the following genes is most likely to
cases. People with mild to moderate
be abnormal?
haemophilia respond to desmopressin
1- Fibrillin-1 sufficiently to cover minor procedures like
2- Collagen Type II tooth extraction. Less than a 2% level is
associated with severe disease, which entails
3- Collagen Type III
spontaneous bleeding into joints and muscles.
4- Collagen Type IV Most cases are the result of X-linked recessive
5- Collagen Type V transmission. Probably one-third of cases may
result from a new mutation. von Willebrand
factor levels are normal.
Answer & Comments
Answer: 1- Fibrillin-1 [ Q: 1338 ] PasTest 2009 - Basic
This man has Marfan’s, his late presentation Science
to medical services perhaps being explained A 17-year-old boy with a history of type 1
by the fact that he is from a travelling family. diabetes mellitus, sensorineural high tone
The history of joint and lens dislocation, and deafness and colour blindess presents for
his Marfanoid habitus is strongly supportive of review in the genetics clinic. You learn from
the diagnosis. Marfan’s occurs due to a defect his mother that other members of the family
in the fibrillin-1 gene, which codes for fibrillin, are affected, including a daughter aged 21
an essential building block of microfibrils, who also suffers from Type 1 diabetes.
which are a component of the suspensory
Which of the following represents the usual
ligament of the lens and other tissues with
mode of inheritance for the likely genetic
elasticity such as the aorta. Collagen gene
syndrome?
defects are associated with Ehlers Danlos
syndrome. 1- Autosomal dominant
2- X-linked
5- Chromosomal non-dysjunction
[ Q: 1340 ] PasTest 2009 - Basic
Science
Answer & Comments
A couple aged in their early 30s come to the
Answer: 4- Autosomal recessive genetics clinic. They have one 6-year-old son
This boy has Wolfram syndrome, a rare who has gross obesity and mild mental
progressive neurodegenerative disorder impairment, diagnosed as Prader Willi
characterised by diabetes insipidus, diabetes syndrome and they are concerned about the
mellitus, optic atrophy and sensorineural possibility of further children being affected.
deafness (DIDMOAD). Patients may also suffer Which of the following most accurately
from red-green colour blindness. describes the mode of inheritance of Prader
Heterozygotes are thought to have an Willi?
increased risk of Type 1 diabetes. Carrier
1- Autosomal recessive
frequency is around 1:350, leading to a
prevalence of 1:770,000. A variety of 2- Autosomal dominant
mutations in the WFS1 gene are thought to be 3- X-linked recessive
responsible.
4- X-linked dominant
5- Non-mendelian
[ Q: 1339 ] PasTest 2009 - Basic
Science
Answer & Comments
Which of the following disorders may have an
autosomal recessive mode of inheritance? Answer: 5- Non-mendelian
1- Achondroplasia Prader Willi occurs because of deletion or
2- Glucose 6-phosphate dehydrogenase disruption of genes on the proximal arm of
deficiency chromosome 15, (15q11-13). It is the first
human disorder to be attributed to genetic
3- Huntington's disease imprinting, where genes are expressed
4- Ehlers-Danlos syndrome Type IV differently based on which parent they came
5- Haemophilia A from. Around 70% of cases occur due to
deletion of 15q11-13, 28% due to maternal
disomy. Prevalence of the condition is
Answer & Comments
estimated as being as high as 1:8000 in rural
Answer: 4- Ehlers-Danlos syndrome Type IV Sweden, and as low as 1:25000 in other areas.
Features include morbid obesity, hyperphagia,
Ehlers-Danlos syndrome is a autosomal hypogonadism and mental impairment.
recessive heritable disorder of connective
tissue with easy bruising, joint hypermobility
[ Q: 1341 ] PasTest 2009 - Basic
(loose joints), skin laxity, and weakness of
Science
tissues. There are a number of different types
of Ehlers-Danlos syndrome (EDS) which share In which of the following genetic diseases is
the foregoing features but can be categorised DNA analysis useful?
into at least nine different types. Type IV may
1- Adult polycystic kidney disease
2- Down's syndrome
Given the history above, what is the most 4- X-linked dominant characteristics
likely mode of inheritance? 5- Mitochondrial characteristics
1- Autosomal dominant
Answer & Comments
2- Autosomal recessive
3- Polysomal Answer: 3- Anticipation
would like to start a family. His partner does 4- Sickle cell disease
not have the disease. 5- Phenylketonuria
What is the percentage that his children will
inherit his disease? Answer & Comments
1- 100%
Answer: 2- α1-antitrypsin deficiency (A1AT)
2- <5%
Alpha-1-antitrypsin deficiency (α1-antitrypsin
3- 25% deficiency, A1AD or Alpha-1) is a genetic
4- 50% disorder caused by defective production of
alpha-1-antitrypsin (A1AT), leading to
5- 0%
decreased A1AT activity in be blood and lungs,
and deposition of excessive abnormal A1AT
Answer & Comments
Answer: 2- <5%
vomiting following the consumption of fish beneficial, which of the following is used for
such as tuna, mackerel and mahi mahi. this indication?
Monosodium glutamate causes abdominal 1- Cyclophosphamide
bloating and vomiting - the so-called ‘Chinese
restaurant syndrome'. Sulphites on prepacked 2- Dapsone
salads causing asthma is called the ‘Salad-bar 3- Methotrexate
syndrome'. 4- Pentoxifylline
5- Thalidomide
[ Q: 1391 ] PasTest 2009 - Basic
Science
Answer & Comments
Selective IgA deficiency is often discovered
incidentally in the investigation of a patient, Answer: 5- Thalidomide
however you will need to explain to the Both methotrexate and pentoxifylline have
patient the clinical significance of such a weak anti-TNF activity, but thalidomide has
finding. clinically significant effects. The problems with
Hence which of the following statements is thalidomide therapy include a potentially
relevant? irreversible neuropathy, teratogenesis,
drowsiness, constipation and weight gain.
1- Predisposition to Staphylococcus aureus
Thalidomide and its derivatives are also used
infection
in the management of leprosy, HIV, myeloma
2- An increased risk of colonic carcinoma and melanoma.
3- IgG2 deficiency leading to recurrent
bacterial infections [ Q: 1393 ] PasTest 2009 - Basic
4- A reduced risk of allergy Science
xanthomas but not of the type described here. attacks, the urine darkens on standing but
Lipoprotein (a) is a modified form of LDL and there is no jaundice. Quinine-like antimalarials
appears to be an independent risk factor for do not precipitate acute attacks of porphyria
coronary disease. in susceptible patients.
known to have diabetes, is found to have a Which of the following types of acid-base
high blood glucose concentration. disturbance is most likely?
Increased secretion of which of the following 1- Acute respiratory acidosis
substances is most likely to be responsible? 2- Chronic, compensated respiratory acidosis
1- Adrenaline (epinephrine) 3- Exacerbation of chronic respiratory acidosis
2- Cortisol 4- Mixed respiratory acidosis and metabolic
3- C-reactive protein alkalosis
4- Growth hormone 5- Severe metabolic acidosis
5- Insulin
Answer & Comments
Answer & Comments Answer: 3- Exacerbation of chronic respiratory
Answer: 1- Adrenaline (epinephrine) acidosis
During the metabolic response to trauma, The patient is acidotic but the elevated
there is increased secretion of bicarbonate concentration is incompatible
catecholamines, cortisol, glucagon and growth with a metabolic acidosis. The high p(CO2)
hormone. The first three of these tend to indicates that there is a respiratory acidosis.
increase blood glucose concentration; Were this to be acute, the bicarbonate
catecholamines, cortisol and glucagon act concentration would be normal or only
directly, whereas growth hormone appears to marginally elevated and a higher hydrogen-ion
potentiate the action of cortisol and opposes concentration would be expected (hydrogen-
the action of insulin. Adrenaline and glucagon ion concentration increases by approximately
act most rapidly, by stimulating 5.5 nmol/l for each increase in p(CO2) by 1
glycogenolysis; cortisol tends to act more kPa in an acute disturbance.) Were
slowly, through the stimulation of compensation to be complete, it would be
gluconeogenesis. Insulin is a hypoglycaemic expected that the patient would be less
hormone. C-reactive protein is a marker of acidotic, and the bicarbonate concentration
inflammation, but does not affect glucose higher.
homeostasis. The degree of the The data are compatible with both an acute
neuroendocrine stress response is related to exacerbation of a chronic respiratory acidosis
the severity of the trauma and the time lapse and a mixed respiratory acidosis and
before treatment (fluid replacement, pain metabolic alkalosis, but the clinical setting
relief etc.) is instituted. Hyperglycaemia can makes the former the more likely.
persist for many hours and even days in some
cases.
[ Q: 1408 ] PasTest 2009 - Basic
Science
[ Q: 1407 ] PasTest 2009 - Basic
Science A 19-year-old man presents with clinical
features suggestive of malabsorption.
A 71-year-old man with known chronic
Which of the following investigations has the
obstructive pulmonary disease is admitted to
potential to indicate a single cause for this?
A&E with severe shortness of breath. Blood
gas analysis shows: arterial [H+] 55 nmol/l (pH 1- Breath hydrogen measurement
7.26), p(CO2) 9.4 kPa, p(O2) 9.1 kPa, derived 2- Faecal fat excretion
[HCO3- ] 31 mmol/l.
3- Lactose tolerance test
elicited after GnRH is given daily for seven most appropriate to provide his energy
consecutive days. requirements?
Which of the following is most likely to be the 1- Dextrose 50% 500 ml, lipid emulsion 20%
cause of his delayed puberty? 500 ml
1- A hypothalamic disorder 2- Dextrose 50% 500 ml, lipid emulsion 20%
2- A pituitary disorder 750 ml
3- paCO2
[ Q: 1418 ] PasTest 2009 - Basic
4- Compliance Science
5- Blood flow
The secretion of growth hormone is increased
by?
Answer & Comments
1- Hyperglycaemia
Answer: 1- V/Q ratio
2- Exercise
The alveoli at the apex of the lung are larger 3- Somatostatin
than those at the base so their compliance is
4- Growth hormone
less. Because of the reduced compliance, less
inspired gas goes to the apex than to the base. 5- Free fatty acids
Also, because the apex is above the heart
level, less blood flows through the apex than Answer & Comments
through the base. However, the reduction in
air flow is less than the reduction in blood Answer: 2- Exercise
flow, so that the V/Q ratio at the top of the Growth hormone (GH) is synthesised, stored,
lung is greater than it is at the bottom. The and secreted by the endocrine cells of the
increased V/Q ratio at the apex makes PaCO2 anterior pituitary. Its release is stimulated by
lower and PaO2 higher at the apex than they growth hormone-releasing hormone and
are at the base . inhibited by somatostatin. Numerous factors
serve as a stimulus for GH release, including
[ Q: 1417 ] PasTest 2009 - Basic hypoglycaemia (e.g. insulin administration),
Science moderate to severe exercise, stress due to
emotional disturbances, illness, and fever, and
The primary neurochemical disturbance in
dopamine agonists such as bromocriptine.
idiopathic Parkinson's disease involves
1- Noradrenaline
[ Q: 1419 ] PasTest 2009 - Basic
2- Dopamine Science
3- Gamma-aminobutyric acid (GABA) A 65-year-old smoker with a history of
4- Substance P transient ischaemic attacks presents with loss
of co-ordination of his right side. On
5- Adrenaline
examination there is decreased skin sensation
in the right half of his face and the left half of
Answer & Comments his body. In addition he has a right Horner
Answer: 2- Dopamine syndrome.
Which vessel is most likely to be involved?
The metabolic alkalosis secondary to a loss of Grade II shock15-30% (750 ml - 1.5 l) blood-
gastric acid is a hypochloraemic alkalosis. volume loss, systolic blood pressure is usually
There is predominant loss of H+ and chloride normal but a tachycardia is present
ions. The alkalosis is perpetuated by the
Grade III shock30-40% (1.5-2 litres) loss,
hypochloraemia, which prevents renal
hypotension, tachycardia and fall in urine
excretion of the excess bicarbonate since its
output seen
proximal tubular reabsorption (with sodium) is
enhanced. Provision of adequate chloride ions Grade IV shock> 40% (> 2 l) blood-volume loss,
allows the excess bicarbonate to be excreted anuria and severe shock observed
and corrects the alkalosis. Dextrose 5%
contains no chloride and dextrose saline [ Q: 1425 ] PasTest 2009 - Basic
contains insufficient for this purpose. Twice Science
normal saline is occasionally used for treating
severe hyponatraemia but has no place in this A 24-year-old man, a new immigrant to the UK
clinical situation. Ringer's lactate is from Eastern Europe, who had worked in his
inappropriate, since the metabolism of the own country as a demolition contractor,
lactate that it contains to bicarbonate would undergoes a medical examination and is found
exacerbate the alkalosis. to have glycosuria. His blood lead
concentration is high.
[ Q: 1424 ] PasTest 2009 - Basic Which of the following findings would suggest
Science that he has a generalised disorder of proximal
tubular function?
You are called to see a 56-year-old man 2 h
after a cardiac catheterisation. He is actively 1- Albuminuria
bleeding from his catheter site and his 2- Hyperkalaemia
dressings and bedclothes are soaked with
3- Increased serum urea concentration
blood.
4- Inability to acidify the urine
Which of the following statements is true?
5- Metabolic acidosis
1- Grade I shock applies with up to a 20% loss
of circulating blood volume
Answer & Comments
2- Loss of 2 litres of blood is consistent with
normal systolic blood pressure Answer: 5- Metabolic acidosis
3- The pulse can remain normal in patients Bicarbonate is normally totally reabsorbed
with grade I shock from the glomerular filtrate in the proximal
4- Anuria is pathognomonic of grade III shock convoluted tubule. Decreased bicarbonate
absorption causes metabolic acidosis (renal
5- Grade IV shock is seen with a 30% loss of
tubular acidosis-type 2); however, the urine
circulating blood volume
can be acidified as the plasma bicarbonate
concentration (and hence the bicarbonate
Answer & Comments load for reabsorption) falls and distal tubular
Answer: 3- The pulse can remain normal in hydrogen-ion secretion is intact. The
patients with grade I shock proteinuria associated with proximal tubular
damage typically affects mainly low molecular
Grade I shockLoss of up to 15% (750ml) of weight proteins that are filtered by the
blood volume; blood pressure is normal but glomerulus. There is no reduction in
there may be a slight tachycardia glomerular filtration and hence no increase in
aetiologies for his renal failure and suggest Answer & Comments
that it is longstanding?
Answer: 1- Rise in pH
1- Anaemia
All the above shift the dissociation curve to
2- Small kidneys on ultrasound examination the right, with the exception of a rise in pH.
3- High serum parathyroid hormone
concentration [ Q: 1430 ] PasTest 2009 - Basic
4- Hypotension Science
5- Hyperuricaemia You have performed a liver biopsy, and shortly
after the procedure the patient develops pain
Answer & Comments on the tip of his right shoulder.
Answer: 2- Small kidneys on ultrasound Which nerve is most likely to be responsible for
examination his pain?
1- Right phrenic nerve
The kidneys are usually decreased in size in
chronic renal failure, except when this is due 2- Axillary nerve
to amyloid or polycystic disease. Kidneys jare 3- Right vagus
usually normal in size in advanced diabetic
4- Right sympatheticus
nephropathy, whereas affected kidneys are
initially enlarged from hyperfiltration. 5- Intercostobrachial nerve
Hypertension can be both a cause and a
consequence of renal failure. Uric acid Answer & Comments
nephropathy can cause renal failure but
hyperuricaemia is usual in chronic renal Answer: 1- Right phrenic nerve
failure, due to decreased excretion. Anaemia The phrenic nerve on both sides originates
(due to decreased erythropoietin secretion) from the ventral rami of thethird to fifth
and secondary hyperparathyroidism cervical nerves. It passes inferiorly down the
(decreased calcitriol secretion causes neck to the lateral border of the scalenus
hypocalcaemia) are features, not causes, of anterior, then it passes medially across the
chronic renal failure. border of scalenus anterior parallel to the
internal jugular vein that lies inferomedially.
[ Q: 1429 ] PasTest 2009 - Basic
The right phrenic nerve pierces the diaphragm
Science
in its tendinous portion just slightly lateral to
The oxygen-haemoglobin dissociation curve is the inferior vena caval foramen. It then forms
shifted to the left by which of the following three branches on the inferior surface of the
factors? diaphragm: anterior, lateral and posterior.
1- Rise in pH These ramify out in a radial manner from the
point of perforation to supply all but the
2- Rise in 2,3-DPG (2,3-diphosphoglycerate) periphery of the muscle.
3- Rise in plasma temperature
4- Rise in blood CO2 content [ Q: 1431 ] PasTest 2009 - Basic
Science
5- Fall in plasma bicarbonate concentration
A patient on enteral nutrition develops
constipation.
What could explain the underlying clinical [ Q: 1433 ] PasTest 2009 - Basic
physiology? Science
1- Hyperosmolar feed You are researching a new agent for the
2- Bacterial contamination management of hypoxia related to acute
pneumonia. Part of the assessment includes
3- Low feed temperature
changes in the pulmonary vasculature in
4- Inadequate fluid replacement response to hypoxia.
5- Reduced intestinal absorptive capacity When considering acute hypoxia, which of the
following is true of the pulmonary
Answer & Comments vasculature?
phenytoin, a drug used for the management [ Q: 1441 ] PasTest 2009 - Basic
of epilepsy, and other anticonvulsants such as Science
sodium valproate, phenobarbital, vigabatrin
A patient has the following urea and
ciclosporin an immunosuppressant drug used electrolytes results: Sodium 140 mmol/l
to reduce organ transplant rejection Potassium 4 mmol/l Chloride 105 mmol/l
Bicarbonate 20mmol/l Calculate the anion
calcium-channel blockers (nifedipine,
gap.
verapamil, diltiazem, oxodipine, amlodipine),
a group of antihypertensive drugs. 1- 19 meq/l
2- 5 meq/l
Other drugs, such as antibiotics
(erythromycin) and hormones, have also been 3- 10 meq/l
also associated with this side-effect. 4- 30 meq/l
5- 0 meq/l
[ Q: 1440 ] PasTest 2009 - Basic
Science
Answer & Comments
A 20-year-old female presents with the
Answer: 1- 19 meq/l
following blood gases: pH 7.48 p(CO2) 3.9
mmHg HCO3- 22 mmol/l H+ 35mmol/l Anion gap = ([Na+] + [K+]) - ([Cl-] + [HCO3-])
What is the most likely reason for these (all units mmol/l).
results? Normal range is 8-16 meq/l
1- Amitriptyline
2- Cushing's syndrome [ Q: 1442 ] PasTest 2009 - Basic
Science
3- Hepatic failure
4- Pregnancy In relation to the nutritional physiology of
patients, which of the following would
5- Diazepam overdose
represent appropriate nitrogen requirements
(g N/kg per day) and calorie requirements
Answer & Comments (kcal/kg per day)?
Answer: 4- Pregnancy 1- Reduced food intake: nitrogen requirement
0.3 g N/kg per day, calorie requirement 35
Respiratory alkalosis results from
kcal/kg per day
hyperventilation which is manifested by
excess elimination of CO2 from the blood and 2- Moderate injury: nitrogen requirement 0.15
a rise in the blood pH. Examples of specific g N/kg per day, calorie requirement 25
causes are listed below: kcal/kg per day
3- Moderate sepsis: nitrogen requirement 0.3
catastrophic central nervous system (CNS)
g N/kg per day, calorie requirement 15
event (CNS haemorrhage)
kcal/kg per day
drugs (salicylates, progesterone) 4- Severe injury: nitrogen requirement 0.3 g
pregnancy (especially the third trimester) N/kg per day, calorie requirement 35
kcal/kg per day
decreased lung compliance (interstitial lung
5- Severe sepsis: nitrogen requirement 0.2 g
disease)
N/kg/day, calorie requirement 15
anxiety kcal/kg/day
P1 receptors are involved in mast cell function 3- Improvement of blood pressure control
and eosinophil apoptosis. 4- Improvement of exercise tolerance
5- Reduction in pain
[ Q: 1444 ] PasTest 2009 - Basic
Science Answer & Comments
Which of the following respiratory physiology Answer: 4- Improvement of exercise tolerance
tests would be consistent with a diagnosis of
moderately established cryptogenic fibrosing Erythropoietin is used as a performance-
alveolitis? enhancing drug. The hormone regulates the
red cell mass in the human body. By increasing
1- Diffusion capacity decreased, FEV1/FVC
red cell mass and therefore the oxygen-
normal, total lung capacity reduced
carrying capacity of the blood, more oxygen is
Investigations: 2- Hypercalcaemia
Prostatitis and acute urinary retention can dilution of the plasma by the excess water,
both result in increased serum PSA but it could also suggest chronic liver disease,
concentrations; digital rectal examination has another cause of chronic hyponatraemia.
a minor, and only transient effect. This level is
certainly compatible with malignancy, but is [ Q: 1475 ] PasTest 2009 - Basic
not diagnostic of it. In general, the higher the Science
PSA, the greater the likelihood of malignancy,
but some patients with malignancy have Which of the following is an acute porphyria?
normal levels (often taken as less than 4 ng/l, 1- Congenital erythropoietic porphyria
but are actually age-dependent). The absolute
2- Erythropoietic protoporphyria
PSA concentration correlates poorly with
prognosis in prostatic cancer. 3- Porphyria cutanea tarda
4- Variegate porphyria
[ Q: 1474 ] PasTest 2009 - Basic 5- None of the above
Science
In a patient with chronic hyponatraemia Answer & Comments
(sodium concentration 112 mmol/l), which of
Answer: 4- Variegate porphyria
the following findings would most suggest a
diagnosis of the syndrome of inappropriate The acute porphyrias include acute
[secretion of] antidiuretic hormone (SIADH)? intermittent porphyria and variegate
1- Normal cortisol response to ACTH porphyria. These are rare genetic errors of
haem biosynthesis. Accumulation of porphyrin
2- Plasma albumin concentration 28 g/l precursors (porphobilinogen and d-
3- Plasma osmolality 248 mOsmol/kg aminolaevulinic acid) cause neuronal and
4- Urinary osmolality 350 mOsmol/kg visceral crises. The acute intermittent type is
inherited as an autosomal-dominant, although
5- Urinary sodium concentration < 20 mmol/l penetrance is low and around one-third of
cases are due to mutations. The urinary
Answer & Comments porphobilinogen level is raised during an acute
attack, but only 50% of samples will turn red
Answer: 4- Urinary osmolality 350 mOsmol/kg
on standing. Faecal levels of porphyrin are
The most significant finding is that the urine is normal in patients with the acute intermittent
concentrated relative to plasma. This is type, unlike the variegate form. Variegate
inappropriate, in that a low plasma osmolality porphyria is also inherited as an autosomal-
should suppress ADH secretion and lead to the dominant and is associated with
formation of a maximally dilute urine. The low photosensitivity causing blistering.
plasma osmolality reflects the hyponatraemia,
since sodium is the principal determinant of [ Q: 1476 ] PasTest 2009 - Basic
extracellular fluid osmolality. A low urine Science
sodium excretion would suggest extrarenal
sodium depletion as a cause of Which of the following findings would most
hyponatraemia. Addison's disease, another suggest that fluid loss from the body was
cause of chronic hyponatraemia, is excluded primarily hypotonic (water depletion) rather
by the cortisol response to ACTH, as it should than isotonic (sodium depletion)?
be before SIADH is diagnosed. The low plasma 1- Hyponatraemia
albumin concentration could just reflect
2- Increased haematocrit
binding at the cell membrane-receptor site glycine or taurine which increases their
results in tyrosine kinase activation and solubility. Intestinal bacteria then convert
initiation of a cascade of intracellular them into secondary bile acids. The bile salt
reactions. One of these is the migration of the pool is small (around 2.5-5 g) and is recycled
GLUT-4 receptor to the cell surface. GLUT-4 is 6-8 times per day via the enterohepatic
the channel through which glucose is taken up circulation, this causes reabsorption of 95% of
into muscle and adipose tissue cells following bile acids. Total faecal loss is around 10-20%
stimulation of the insulin receptor. of the bile acid pool in any one day.
capillary endothelium and for triglycerides to (the most frequent presenting feature of the
be progressively removed by lipoprotein condition), Paget's disease is the more likely.
lipase. This leaves a particle depleted of
triglyceride and apoprotein C-II, known as an [ Q: 1493 ] PasTest 2009 - Basic
IDL (intermediate-density lipoprotein) particle. Science
Most familial hypertriglyceridaemia is A 36-year-old man presents to his family
polygenic in origin, and due to a modest doctor complaining of excessive sweating.
excess in the circulating concentration of VLDL Thyroid function tests are performed: serum
particles. There may be a history of retinal TSH concentration is normal, but the
vein thrombosis or recurrent pancreatitis in concentrations of both free thyroxine and free
individuals affected by the familial condition. triiodothyronine are elevated.
Which of the following is the most likely
[ Q: 1492 ] PasTest 2009 - Basic explanation for these results?
Science
1- A TSH-secreting pituitary tumour
A 71-year-old-man is given a health check by
2- Graves' disease
his doctor. He has no complaints, apart from
pain in his hip, which he attributes to arthritis. 3- Self-administration of thyroxine
He has blood taken for a panel of routine 4- The presence of heterophilic antibodies in
biochemical and haematological tests. The the patient's serum
only abnormality is a serum alkaline
5- Thyroid hormone resistance
phosphatase activity of 822 U/l.
Which of the following is the most likely cause
Answer & Comments
of this abnormality in his case?
Answer: 1- A TSH-secreting pituitary tumour
1- Metastatic carcinoma
2- Osteoarthritis These results are unusual: patients with
thyrotoxicosis from, for example, Graves'
3- Osteomalacia
disease would be expected to have
4- Osteoporosis unmeasureable TSH concentration (the high
5- Paget's disease concentrations of thyroid hormones should
suppress TSH secretion). Heterophilic
antibodies (antibodies in the patient's serum
Answer & Comments
that react with animal-derived antibodies
Answer: 5- Paget's disease used in the immunoassay) can cause bizarre
results, but this patient has a classic symptom
Elevated alkaline phosphatase activity is
of thyrotoxicosis. Because of this, thyroid
usually an indicator of bone or cholestatic
hormone resistance is also unlikely. If he were
hepatobiliary disease. The fact that other liver
self-administering thyroid hormones, TSH
function tests are normal militates against
secretion should be suppressed.
(though does not exclude) the latter. An
increase in alkaline phosphatase from bone TSH-secreting pituitary tumours are
reflects increased osteoblastic activity: this is uncommon, but are a recognised cause of
not a feature of osteoporosis or osteoarthritis hyperthyroidism. They sometimes co-secrete
and alkaline phosphatase is not elevated in growth hormone, excessive secretion of which
these conditions. Increases are seen in is also a cause of sweating.
metastatic bone disease and in Paget's, but
given that the patient is well apart from pain
Which of the following clinical features, if XYY syndrome. The combination of tall
present, would most direct you towards a stature, gynaecomastia and infertility due to
specific cause? azoospermia is diagnostic of Klinefelter's
1- Bone pain syndrome. Almost all males with Klinefelter
syndrome are azoospermic. Other features of
2- Hilar lymphadenopathy this condition include reduced facial hair,
3- Polyuria tendency to obesity, and small atrophic testes.
Mild learning difficulties can be seen in some
4- Short QT interval
patients with Klinefelter's syndrome but
5- Ureteric colic mental retardation is not a feature of this
condition. Chromosome analysis will show 47
Answer & Comments chromosomes, with two X and one Y
chromosome (47, XXY).
Answer: 2- Hilar lymphadenopathy
Patients with Homocystinuria have tall
Bone pain can occur with hypercalcaemia
stature, learning difficulties, lens dislocation
secondary to malignancy or
osteoporosis and recurrent arterial
hyperparathyroidism. Polyuria is a feature of
thrombosis. Features of Marfan's syndrome
severe hypercalcaemia, irrespective of the
include tall stature, arachnodactyly, scoliosis,
cause. A short QT interval is also a feature of
joint laxity, lens dislocation, aortic root
hypercalcaemia. Ureteric colic is particularly
dilatation, dural ectasia, skin striae, and
associated with primary hyperparathyroidism,
recurrent pneumothorax. The XYY syndrome is
but is not specific to this cause. The presence
characterised by tall stature, mild learning
of hilar lymphadenopathy in a patient with
difficulties and behavioural problems. Most
hypercalcaemia should raise a suspicion that
males with this condition have normal fertility,
the latter is due to sarcoid (in which the
although a small proportion of patients will
granulomas secrete calcitriol, 1,25-
have infertility due to azoospermia.
dihydroxycholecalciferol).
Arcus senilis, periorbital xanthelasmas and found. Magnetic resonance imaging shows
tendon xanthomas can all occur in patients changes in the central nervous system, but the
with hypercholesterolaemia, although arcus longer term clinical implications are uncertain.
senilis and, less frequently, xanthelasmas, can
Plasma ornithine values range from 400 to
occur in individuals with normal plasma
1000 mmol/l (normal 75 m mol/l) with high
cholesterol concentrations. Tendon
concentrations in cerebrospinal fluid and the
xanthomas (particularly Achilles tendons and
aqueous humour. Between 400 and 900
extensor tendons on the back of the hand) are
mg/day is excreted with increased amounts of
specific to familial hypercholesterolaemia.
arginine and lysine (competitive inhibition of
Eruptive xanthomas are characteristic of
reabsorption). The activity of ornithine-d-
severe hypertriglyceridaemia; palmar
aminotransferase is low in liver and skeletal
xanthomas (typically in the palmar creases,
muscle. Most affected patients have less than
but sometimes more extensive) are typical of
1% of the normal activity in fibroblasts.
familial dysbetahyperlipoproteinaemia
(remnant dyslipidaemia, broad beta disease). The clinical picture and the amino acid defects
are an adequate means of diagnosis. Enzyme
[ Q: 1499 ] PasTest 2009 - Basic assays can be used to confirm the diagnosis.
Science
[ Q: 1500 ] PasTest 2009 - Basic
A 19-year-old patient presents with gradual
Science
worsening myopia and decreased night vision.
On examination there is atrophy of the retina. Antibodies to which of the following are most
What enzyme deficiency is this patient most frequently present in the serum of patients
likely to have? with type-1 diabetes at diagnosis?
causal relationship between infection and the [ Q: 1502 ] PasTest 2009 - Basic
development of diabetes. There is an Science
association between congenital rubella and
A 35-year-old woman is referred by her GP
type-1 diabetes, but the basis of the
because she has been feeling unwell and has
association is unclear.
pain, stiffness and tenderness in her joints
which have gradually worsened over the last 6
[ Q: 1501 ] PasTest 2009 - Basic months. She has a detectable rheumatoid
Science factor.
A 25-year-old patient presents with yellow What is the most likely structure that these
papules on the extensor surfaces of the arms, antibodies are targeted against?
legs, buttocks and back. His triglycerides are
1- Double-stranded DNA
grossly elevated. On questioning he admits
that there is a family history. 2- Tumour necrosis factor-alpha
reactive astrocytes but without abnormal What is the most likely pathological
neurones. mechanism?
1- Bicarbonate loss due to ascites
[ Q: 1506 ] PasTest 2009 - Basic
2- Reduced urea synthesis
Science
3- Increased gastric acid production
You suspect that a 48-year-old man is
4- Reduced bicarbonate secretion from the
suffering from Cushing's syndrome.
pancreas
At what time of day is a random cortisol test
most likely to be abnormal? 5- Reduced lactate formation in skeletal
muscle
1- 0900 h
2- 1200 h Answer & Comments
3- 2400 h Answer: 2- Reduced urea synthesis
4- 1700 h
Urea production is an important feature of
5- 2000 h hepatic metabolism. The production of each
molecule of urea (ultimately from ammonium
Answer & Comments and carbon dioxide) is accompanied by the
generation of two protons. Ureagenesis is
Answer: 3- 2400 h
therefore a potential acidifying mechanism.
Plasma cortisol levels in normal individuals Most of the protons produced in ureagenesis
show a circadian rhythm. Levels are highest in are neutralised by the bicarbonate generated
the early morning, and fall to their lowest during the oxidation of the carbon skeleton of
levels during sleep at around midnight. In amino acids. Normally, however, a slight
patients with Cushing's syndrome the 0900 h excess of protons is produced that has to be
cortisol level may be within the normal range, eliminated by the kidneys.
but midnight levels are usually not
Urea synthesis and accompanying proton
suppressed. However, isolated venous blood
production are negatively regulated by
tests for cortisol levels are not the best way to
acidosis, which constitute another acid-base
detect Cushing's syndrome. A 24-h urine
regulatory system intrinsic to the liver.
collection with analysis of cortisol levels is a
much better early screening test for Cushing's.
[ Q: 1508 ] PasTest 2009 - Basic
A reverse circadian rhythm is seen for Science
melatonin, with levels at their highest during
periods of sleep. Synthetic melatonin is widely A 34-year-old immigrant of African origin is
used by business travellers for relief of ‘jet- prescribed ciprofloxacin for an infection. He
lag'. The best example of a longer and more suffers problems with acute haemolysis. You
complex human biological rhythm is the 28- suspect G6PD deficiency.
day menstrual cycle. Given this, what is the likely underlying cause
of haemolysis?
[ Q: 1507 ] PasTest 2009 - Basic 1- Increased levels of NADPH
Science
2- Decreased levels of NADP
A patient with liver cirrhosis develops 3- A defect on chromosome 29
metabolic alkalosis.
4- Reduced levels of ATP
recurrent attacks suggest the hereditary small left homonymous hemianopia with
cause. C1 esterase inhibitor concentrate or partial sparing of central vision.
fresh-frozen plasma is used to treat Which of the following is the most likely
recalcitrant cases. anatomical site of the lesion responsible?
1- Right occipital cortex
[ Q: 1515 ] PasTest 2009 - Basic
Science 2- Left occipital cortex
Retinal lesions result in a paracentral scotoma, Brainstem lesions involving the fifth nerve
mononuclear field loss results from an optic nuclei may include brainstem glioma, multiple
nerve lesion. Lesions at the chiasm cause sclerosis, brainstem infarction or
bitemporal hemianopia (seen with pituitary syringobulbia. Lesions at the cerebellopontine
macroadenomas, for example). Proximal optic angle resulting in fifth nerve damage may
tract lesions result in a homonymous include acoustic neuroma, meningioma and
hemianopia. More distal upper lesions secondary tumour deposits. Within the
resulting from temporal lobe damage result in cavernous sinus, the trigeminal ganglion may
an upper homonymous quadrantanopia, be compressed by a pituitary tumour
parietal lobe damage results in a lower extending into the sinus, internal carotid
homonymous quadrantanopia. Widespread artery aneurysm, cavernous sinus thrombosis
diffuse bilateral occipital lobe damage may or secondary tumour. The trigeminal ganglion
cause cortical blindness, the so-called ‘Anton's may also be affected by herpes zoster
syndrome'. This unfortunate condition results infection. Prognosis for the recovery of
in patients having an inability to see, but they trigeminal nerve function is dependent on the
have little or no insight into the fact. underlying cause.
through the temporal and parietal lobes, the be upper motor neurone weakness. Sensory
optic radiations separate, with the lower root compression typically causes pain in the
bundle of fibres (carrying information from dermatome supplied by the root; peripheral
the superior visual fields) passing through the polyneuropathy is usually bilateral.
temporal lobe, and upper fibres (carrying
information from the inferior visual fields) [ Q: 1526 ] PasTest 2009 - Basic
going through the parietal lobe. Thus, at these Science
anatomical sites a quadrantanopia is a
possible consequence of localised damage. A A 24-year-old woman attends the neurological
temporal lobe lesion produces a contralateral clinic for review of multiple sclerosis,
superior homonymous quadrantanopia, and a diagnosed 2 years before. She had presented
parietal lobe lesion (as in this case) produces a with blurring of vision and mild pain in her left
contralateral inferior homonymous eye, which had resolved over a period of 3
quadrantanopia. This corresponds to lesion 5 months and had not recurred. On examination
as shown on the diagram below. now, the following observations are made:
light shone in the left eye causes constriction
of the left and right pupils; light shone into the
[ Q: 1525 ] PasTest 2009 - Basic
right eye causes constriction of the right and
Science
left pupils but when the light is shone back
A 36-year-old man presents to his GP with a into the left eye, the left pupil dilates slightly.
feeling of numbness in his left leg. On Which of the following is the most likely site of
examination, he has decreased position sense the lesion responsible?
and light touch and vibration sensation
affecting his left leg to the upper part of the 1- Left ciliary ganglion
thigh. No other neurological deficit is 2- Left oculomotor nerve
demonstrable.
3- Left optic nerve
Which of the following is the most likely cause
4- Right ciliary ganglion
of this presentation?
5- Right optic nerve
1- Left dorsal column lesion
2- Left spinothalamic tract lesion Answer & Comments
3- Peripheral polyneuropathy
Answer: 3- Left optic nerve
4- Partial section of the spinal cord
The abnormal response is on the left; the right
5- Sensory root compression pupil shows a normal direct and consensual
response. The normal constriction of the left
Answer & Comments pupil indicates that the efferent pathway
(involving the Edinger-Westphal nucleus,
Answer: 1- Left dorsal column lesion
oculomotor nerve and ciliary ganglion) is
The dorsal columns carry ipsilateral intact. The defect is in the afferent pathway
proprioreception, light touch and vibration on the left, which involves fibres in the optic
sensation. Spinothalamic tract lesions cause nerve. The sign demonstrated is a relative
contralateral loss of pain and temperature afferent pupillary defect (RAPD), implying
sensation. Partial section of the cord tends to partial damage only to the afferent pathway:
cause contralateral pain and temperature if all function is lost, neither the direct reflex
sensation and ipsilateral loss of the modalities nor consensual reflex (constriction of the right
carried in the dorsal columns; there may also
pupil in response to light shone into the left 2- The normal portal vein pressure is 8-12
eye) will be present. mmHg
3- The portal vein supplies 75% of liver blood
[ Q: 1527 ] PasTest 2009 - Basic flow
Science
4- The caudate lobe of the liver does not have
You review a 39-year-old sportsman who it's own branch of the hepatic vein
complains of knee pain. Arthroscopy reveals 5- Only the portal vein enters the liver via the
damage to the cartilage. porta hepatis
Which of the following stems best describes a
property of hyaline cartilage? Answer & Comments
1- It has a blood supply from small arterioles Answer: 3- The portal vein supplies 75% of
2- It is rich in type 1 collagen liver blood flow
3- Chondrocytes secrete collagen only The blood supply of the liver is around 25% of
4- It is avascular the resting cardiac output and arises from two
main vessels. The hepatic artery, a branch of
5- Pressure from normal joint loading
the coeliac axis supplies 25% of the total liver
accelerates damage to cartilage
blood flow. Autoregulation of blood flow by
the hepatic artery ensures constant liver
Answer & Comments blood flow. The portal vein drains most of the
Answer: 4- It is avascular gastrointestinal (GI) tract and the spleen and
constitutes 75% of liver blood flow. Normal
Hyaline cartilage forms the articular surface portal pressure is 5-8 mmHg, but blood flow
and is avascular, relying on diffusion from increases after meals. Both vessels enter the
synovial fluid for nutrients. It is rich in type II liver via the porta hepatis. The caudate lobe
collagen and forms a meshwork containing receives an independent blood supply from
proteoglycan molecules that retain water. the hepatic portal vein and artery and its
Intermittent pressure from joint loading is branch of the hepatic vein drains directly into
essential to maintain normal cartilage the inferior vena cava.
function. Chondrocytes secrete proteoglycans
and collagen and are embedded in the
[ Q: 1529 ] PasTest 2009 - Basic
cartilage. They migrate to the joint surface
Science
along with the matrix that they produce.
You are asked to see a 45-year-old man who is
[ Q: 1528 ] PasTest 2009 - Basic haemodynamically compromised and plan to
Science insert a right subclavian line.
He has a body mass index (BMI) of 38, where
You review a 54-year-old man with a history of
is the correct position for central venous
alcoholism; you are concerned that there may
cannulation?
be evidence of portal hypertension.
1- 1 cm under the mid-point of the clavicle
Which of the following stems best describes
and 0.5 cm laterally
blood flow to the liver?
2- 2 cm under the mid-point of the clavicle
1- The hepatic artery supplies 75% of the total
and 1 cm laterally
liver blood flow
3- 2.5 cm under the mid-point of the clavicle
and 2 cm laterally
4- 0.5 cm under the mid-point of the clavicle This patient has acromegaly. Had the illness
and 1 cm laterally started before epiphyseal fusion, he would
5- 1 cm under the mid-point of the clavicle have had gigantism. Measuring the level of
and 1 cm laterally basal growth hormone is not an appropriate
diagnostic test because of variations in
concentration, although levels are often high.
Answer & Comments
Biochemical diagnosis is made by measuring
Answer: 2- 2 cm under the mid-point of the growth-hormone levels in response to a
clavicle and 1 cm laterally glucose challenge. In normal patients, the
concentration of growth hormone should fall.
In obese patients, the standard position for Prolactin is structurally similar to growth
right subclavian central venous cannulation is hormone. Levels of prolactin increase during
2 cm under the mid-point of the clavicle and 1 sleep and in response to estrogen and stress,
cm laterally. In thin patients the standard as well as during suckling in breast-feeding
position for insertion is 1 cm under the mid- women.
point of the clavicle and 0.5 cm laterally.
Jugular vein cannulation is now the preferred
[ Q: 1531 ] PasTest 2009 - Basic
choice for central venous catheterisation, as
Science
insertion under ultrasound guidance is
associated with a much lower rate of Intervertebral disc prolapse in the lumbar
complications than subclavian insertion. The spine most often affects the L4/l5 and L5/S1
major hazard of the subclavian approach is discs.
arterial puncture, as the artery lies close to
In a man presenting with acute back pain
the vein.
following an episode of lifting a heavy weight,
reduced force of which of the following
[ Q: 1530 ] PasTest 2009 - Basic movements would most suggest an L5/S1 (L5
Science root) rather than an L4/5 disc lesion (L4 root)?
A 45-year-old man presents with prognathism 1- Ankle plantar flexion
and interdental separation.
2- Eversion of the foot
Which of the following is the most appropriate
3- Extension of great toe
investigation?
4- Inversion of the foot
1- Fasting glucose test including growth
hormone measurement 5- Knee extension
4- Serum prolactin measurement In the lumbar spine (in contrast to the cervical
5- Thyroid function test spine) nerve roots emerge below their
respective vertebrae: thus an L4/5 disc lesion
would be expected to affect the L4 root and
Answer & Comments
an L5/S1 disc lesion to affect the L5 nerve
Answer: 2- Glucose tolerance test including root. Knee extension is mediated by L2, 3 and
growth hormone measurement 4; ankle dorsiflexion by L4 and 5; inversion of
the foot by L4 alone; eversion of the foot by
S1, and ankle plantar flexion by S1 and 2.
Although L5 contributes to hip abduction and The internal capsule comprises an anterior
extension, knee flexion and ankle dorsiflexion, limb, between the head of the caudate
weakness in the clinical situation of L5 root nucleus and the lentiform nucleus, and the
compression is often minimal because of the posterior limb between the latter and the
contribution of other roots to these thalamus. The junction between the limbs is
movements. Weakness tends to be maximal in the genu, (‘knee'). The lentiform nucleus itself
extension of the toes, particularly the great comprises an outer putamen and an inner
toe. This is due to impairment of extensor globus pallidus. Infarction of this region is
hallucis longus, a branch of the deep peroneal often due to occlusion of a perforating branch
nerve (origin L4, 5 and S1 but predominantly of the middle cerebral artery.
L5 for extensor hallucis longus), which
contributes to ankle dorsiflexion and is [ Q: 1533 ] PasTest 2009 - Basic
responsible for dorsiflexion of the great toe. Science
A 50-year-old woman has a suspected right
[ Q: 1532 ] PasTest 2009 - Basic
renal artery stenosis. A transfemoral
Science
aortogram has been performed.
A 52-year-old hypertensive man presents with Which one of the following is likely to be
a right hemisphere stroke. Neuroimaging correct?
demonstrates the presence of a recent
infarction of the right internal capsule. 1- The femoral artery at the groin is situated
halfway between the anterior superior iliac
Which anatomical features of this region spine and the pubic tubercle
should be borne in mind?
2- The catheter passes through the common
1- The anterior limb of the internal capsule lies femoral artery into first the external iliac
between the tail of the caudate nucleus artery and then the aorta at its bifurcation
and the lentiform nucleus
3- The right renal artery also gives off the right
2- Medial to the posterior limb lies the septum ovarian and suprarenal arteries
lucidum
4- The right and left renal arteries lie in the
3- The lentiform nucleus itself comprises an transpyloric plane at the level of the first
outer globus pallidus and an inner lumbar vertebra
putamen
5- The aorta passes through the diaphragm at
4- The junction of the anterior and posterior the level of the tenth thoracic vertebra
limbs of the internal capsule is termed ‘the
crus'
Answer & Comments
5- The internal capsule receives its arterial
supply from the lenticulostriate vessels Answer: 4- The right and left renal arteries lie
derived from the roots of the middle and in the transpyloric plane at the level of the
anterior cerebral arteries first lumbar vertebra
[ Q: 1534 ] PasTest 2009 - Basic Which signs are most likely to be present on
clinical examination?
Science
1- There is ptosis on the right side
A patient presents with mononeuritis
multiplex affecting the oculomotor nerve (III). 2- The pupil on the right side is constricted
and fails to respond to light
What clinical feature is likely to be present on
examination? 3- The right eyelid is numb
1- Ptosis of the upper eyelid on the affected 4- The patient is unable to deviate his right
side eye medially
2- Constricted pupil on the affected side 5- The patient is unable to deviate his right
eye laterally
3- Inability to laterally deviate the eye on that
side
Answer & Comments
4- Decreased sweating of the face on the
affected side Answer: 5- The patient is unable to deviate his
right eye laterally
5- A light shone into the affected eye fails to
produce constriction of the pupil on the The abducent nerve innervates the lateral
opposite side rectus muscle of the eye exclusively; the sole
effect of damage to this nerve is that the
Answer & Comments patient is unable to abduct (laterally deviate)
the eye.
Answer: 1- Ptosis of the upper eyelid on the
affected side
[ Q: 1536 ] PasTest 2009 - Basic
The oculomotor nerve supplies the levator Science
palpebrae superioris - paralysis of which
A dental surgeon carries out a block of the
results in ptosis (lid lag). It supplies
inferior alveolar nerve by infiltrating local
parasympathetic fibres to the constrictor
anaesthetic at the mandibular foramen.
pupillae, so the pupil widely dilates if this
nerve is damaged. The nerve supplies all the Which clinical feature may result from this
extrinsic muscles of the eyeball apart from the procedure?
lateral rectus (abducent nerve, VI) and 1- Numbness of the lower lip on the injected
superior oblique (trochlear nerve, IV). side
Abduction of the eye is thus retained.
2- Ineffective block for the incisor teeth
The nerve plays no part in the supply to the 3- Numbness of the side of the tongue
facial skin (trigeminal nerve, V), and it is the
optic nerve (II) that mediates the pathway of 4- Inability of the patient to clench his jaws
pupillary constriction to the opposite eye 5- Transient weakness of the facial muscles on
when a light is shone into the eye on the the injected side
affected side.
[ Q: 1539 ] PasTest 2009 - Basic The sinus node is supplied by the right
Science coronary artery in around 60% of people, the
AV node in around 90%.
You are reviewing a 52-year-old man who has
suffered a myocardial infarction. You suspect
occlusion of the posterior descending [ Q: 1540 ] PasTest 2009 - Basic
coronary artery. Science
communicate with the anterior facial vein, year-old woman, following a suspected
which drains the face and upper lip - hence aneurysmal bleed.
the danger of spread of infection from this Which anatomical feature should be
locus. considered when interpreting the angiogram?
1- The middle cerebral artery is the largest
[ Q: 1543 ] PasTest 2009 - Basic
single component of the circle of Willis
Science
2- The posterior cerebral artery is clearly seen
A motor cyclist involved in a road traffic on a lateral carotid angiogram
accident sustained an injury to the brachial
plexus on the right side. He is found to have 3- The vertebral arteries meet at the foramen
weakness of right shoulder abduction and magnum to form the basilar artery
forearm flexion, as well as some sensory loss 4- The middle cerebral artery courses over the
over the lateral aspect of his upper arm. The lateral aspect of the temporal lobe of the
right biceps and brachioradialis reflexes are cerebrum
absent. 5- The middle meningeal artery is an
What is the likely level of maximal plexus extracranial branch of the internal carotid
injury? artery
1- C4,5 root
Answer & Comments
2- C5,6 root
3- C6,7 root Answer: 1- The middle cerebral artery is the
largest single component of the circle of Willis
4- C7,8 root
The middle meningeal artery is a branch of the
5- C8, T1 root
maxillary artery, one of the terminal branches
of the external carotid. The posterior cerebral
Answer & Comments artery arises from the termination of the
Answer: 2- C5,6 root basilar artery, which itself arises from the two
vertebral arteries that meet on the under
A C5/C6 lesion, Erb's palsy, produces sensory surface of the brainstem - so the posterior
loss over the lateral aspect of the upper arm cerebral artery can only be visualised by
(deltoid paralysis), with loss of shoulder vertebral angiography. The middle cerebral
abduction, and paralysis of the biceps, artery is indeed the largest component of the
brachialis and coracobrachialis. In addition to circle of Willis, being, in effect, the
loss of elbow flexion, the biceps is also a termination of the internal carotid. It passes
powerful supinator of the forearm, so the through the lateral sulcus of the cerebrum
forearm assumes a pronated position. A T1 between the temporal and frontal lobes.
lesion produces a claw hand, (Klumke's palsy).
Sympathetic chain injury results in a Horner's
[ Q: 1545 ] PasTest 2009 - Basic
syndrome, with ptosis of the upper eyelid and
Science
constriction of the pupil (meiosis) on the
affected side. A hypertensive, heavy smoking, 73-year-old
man suffers a massive cardiac infarct following
[ Q: 1544 ] PasTest 2009 - Basic occlusion of his anterior interventricular
Science artery, (anterior descending artery).
Angiography is performed to demonstrate the
A cerebral angiogram is performed on a 37- coronary vessels.
Which anatomical relationship of these vessels 3- The transverse (horizontal) fissure of the
should be borne in mind? right lung corresponds to the right fifth
1- The anterior interventricular artery arises intercostal space
above the left posterior aortic cusp 4- The lower border of the lung on each side
2- The anterior interventricular artery supplies corresponds to the tenth rib in the mid-
almost all of the left ventricle axillary line
3- There is a rich collateral circulation 5- The lower border of the lung reaches the
between the right and left coronary twelfth rib posteriorly
arteries
Answer & Comments
4- The circumflex artery is the major branch of
the right coronary artery Answer: 2- The oblique fissure of the lung
5- The posterior interventricular artery is a corresponds to the medial border of the
branch of the circumflex artery scapula when the arm is fully abducted
to tight control, and a high intake does not and is also poorly tolerated. Fibrates are
cause hypercalcaemia. The two most common effective in combination with statins;
causes of hypercalcaemia are primary however, there is a theoretical increased risk
hyperparathyroidism and malignancy. In an of myositis when the drugs are used in
asymptomatic individual, primary combination, although this is not a problem in
hyperparathyroidism is the more likely cause. practice. A fibrate might be appropriate if the
triglyceride concentration is elevated or the
[ Q: 1550 ] PasTest 2009 - Basic HDL-cholesterol is low, but the treatment of
Science choice would be to add ezetimibe, an inhibitor
of cholesterol absorption from the gut.
A patient with familial hypercholesterolaemia
has a total cholesterol concentration of 10.2
[ Q: 1551 ] PasTest 2009 - Basic
mmol/l, LDL-cholesterol 8.1 mmol/l, HDL-
Science
cholesterol 1.2 mmol/l and fasting
triglycerides 1.9 mmol/l. He has a strong A 19-year-old man is admitted by ambulance
family history of premature myocardial after falling into a river. He is pulled out by
infarction. He is a non-smoker and two friends but is thought to have inhaled a
normotensive. He is given lifestyle and dietary significant quantity of water. On examination
advice and prescribed a statin. Some 2 months in the Emergency room his saturation is 90%
after being on the maximum dose of the on O2 by mask. He is drowsy but conscious,
statin, his total cholesterol concentration is with bradycardia and a temperature of 34.8oC.
6.8 mmol/l, LDL-cholesterol 5.2 mmol/l, HDL- Auscultation of his chest reveals wheeze and
cholesterol 1.3 mmol/l and fasting crackles consistent with fluid inhalation.
triglycerides 1.0 mmol/l. Which of the following is the most likely
What would be the most appropriate next step biochemical imbalance to be seen?
in his management? 1- Alkalosis on ABG measurement
1- Adding a bile-acid sequestrant to his 2- Low Urea
medication
3- Acidosis on ABG measurement
2- Adding a fibrate to his medication
4- Hypokalaemia
3- Adding ezetimibe to his medication
5- Hyponatraemia
4- Adding nicotinic acid to his medication
5- Continuing on the present medication with Answer & Comments
review in a further 2 months
Answer: 3- Acidosis on ABG measurement
Answer & Comments Intravascular volume depletion is common
whatever type of fluid has been aspirated. As
Answer: 3- Adding ezetimibe to his medication
such a low measured urea, sodium or
The effects of statins are usually maximal by 4 potassium is unlikely. Hypoxaemia and
weeks after an increase in dose, and a further acidosis are the most common findings apart
fall in cholesterol is unlikely to occur if the from volume depletion. Treatment of near
dose was increased 2 months ago. Bile-acid drowning centres on appropriate re-warming,
sequestrants are effective cholesterol- fluid resuscitation and correction of hypoxia.
lowering agents but are poorly tolerated 35-60% of patients with near drowning die
because of gastrointestinal side-effects. either en route to hospital or in the
Nicotinic acid often causes cutaneous flushing, Emergency department. Because of
A 42-year-old man with a history of road Answer: 5- They contain apolipoprotein B-100
traffic accident and injury to his back and neck The major fat in low-density lipoproteins is
presents with global muscle wasting of the left cholesterol, but the concentration is
hand. determined mainly by the rates of hepatic
Which is the nerve or nerve root most likely to synthesis and peripheral clearance; dietary
be involved? saturated fat intake correlates more strongly
1- Radial nerve with LDL-cholesterol concentration than
dietary cholesterol intake. LDLs are formed
2- Median nerve from VLDL (very-low-density lipoprotein) and
3- Ulnar nerve IDL (intermediate-density lipoprotein) by
successive removal of triglyceride and
4- T1 nerve root
modification in the circulation. It is HDL-
5- C7 nerve root cholesterol that is involved in reverse
cholesterol transport. Each particle of LDL
Answer & Comments contains one molecule of apolipoprotein B-
100, the ligand for the LDL receptor.
Answer: 4- T1 nerve root
intravenous fluid replacement. Her serum most frequently associated with which one of
calcium concentration is 1.82 mmol/l, the following?
phosphate 1.28 mmol/l, alkaline phosphatase 1- Carcinoid tumours
82 U/l (normal < 150), albumin 30 g/l,
creatinine 80 m mol/l. Prior to surgery, her 2- Lymphoma
serum calcium concentration was 2.18 3- Multiple myeloma
mmol/l, albumin 36 g/l. 4- Small-cell carcinoma of the bronchus
What is the most likely cause of her 5- Squamous-cell carcinoma of the bronchus
hypocalcaemia?
1- Formation of insoluble calcium salts in the Answer & Comments
intestine
Answer: 5- Squamous-cell carcinoma of the
2- Hypoalbuminaemia
bronchus
3- Hypomagnesaemia
Hypercalcaemia is a frequent complication of
4- Malabsorption of calcium cancer, and is most usually the result of the
5- Malabsorption of vitamin D secretion of parathyroid hormone-related
peptide. Squamous-cell carcinomas are
Answer & Comments particularly frequently responsible, but
hypercalcaemia is uncommon with other
Answer: 3- Hypomagnesaemia bronchogenic cancers. In myeloma, the
Impaired fat absorption can lead to the secretion of osteoclast-activating cytokines is
formation of insoluble calcium salts in the gut. the usual cause of hypercalcaemia, and in
Fat and calcium are absorbed in the proximal lymphomas, unregulated extrarenal
small intestine, so, too, is vitamin D. Although production of calcitriol can be responsible.
bile salts are absorbed distally, and impaired Carcinoid tumours sometimes secrete
absorption can lead to a secondary decrease vasopressin or ACTH, but rarely cause
in proximal fat absorption, this is unlikely to hypercalcaemia.
be responsible for hypocalcaemia developing
so quickly. The normal alkaline phosphatase [ Q: 1561 ] PasTest 2009 - Basic
level also militates against vitamin D Science
deficiency. Hypocalcaemia would normally be
Osteoporosis is most reliably diagnosed by
expected to stimulate parathyroid hormone
which one of the following techniques?
secretion and cause the plasma phosphate
concentration to fall (PTH is phosphaturic). 1- Dual-energy X-ray absorptiometry (DEXA)
Patients with ileostomies can lose large 2- Measurement of serum osteocalcin
amounts of magnesium through their stomas;
3- Measurement of urinary collagen
hypomagnesaemia impairs PTH secretion and
telopeptides
can cause hypocalcaemia that is resistant to
an increased provision of calcium. 4- Quantitative computed tomography (CT)
5- Quantitative ultrasonography
[ Q: 1560 ] PasTest 2009 - Basic
Science Answer & Comments
Hypercalcaemia in malignant disease Answer: 1- Dual-energy X-ray absorptiometry
secondary to the secretion of parathyroid (DEXA)
hormone-related peptide by the tumour is
DEXA is currently the most precise and [ Q: 1563 ] PasTest 2009 - Basic
accurate method for diagnosing osteoporosis. Science
Quantitative CT of the lumbar spine is less
A 48-year-old man is referred by his GP with
precise (and more expensive, and involves a
suspected acromegaly.
greater exposure to radiation). Quantitative
ultrasound measurements on the calcaneum Which of the following would be the most
are being evaluated but have not been proven useful investigation to establish the diagnosis?
to be superior to DEXA. Biochemical indices of 1- Glucose tolerance test with measurement
bone formation (eg osteocalcin) and of growth hormone
breakdown (N- and C-terminal cross-linking
2- Measurement of serum basal growth
telopeptides of type-1 collagen) are of value in
hormone concentration
monitoring the response of patients to
treatment, but not for diagnosis. 3- Measurement of serum growth hormone
concentration during sleep
[ Q: 1562 ] PasTest 2009 - Basic 4- Measurement of serum insulin-like growth
Science factor I (IGF-1, somatomedin C)
Which of the following features most reliably 5- Measurement of serum growth hormone
suggests that a patient presenting with concentration following exercise
diabetes has type 1?
1- Family history of diabetes Answer & Comments
An azygous lobe is seen in about 0.5% of A small area along the upper part of the
routine chest X-rays and is a normal variant. It medial border of the left kidney is in relation
is seen as a ‘reverse comma sign' behind the with the left suprarenal gland, and close to the
medial end of the right clavicle. lateral border is a long strip in contact with
the renal impression on the spleen. A
[ Q: 1567 ] PasTest 2009 - Basic somewhat quadrilateral field, about the
Science middle of the anterior surface, marks the site
of contact with the body of the pancreas, on
Which of the following structures is located in the deep surface of which are the lienal
the anterior mediastinum on computed vessels. Above this is a small triangular
tomography (CT)? portion, between the suprarenal and splenic
1- Thymus areas, in contact with the postero-inferior
surface of the stomach. Below the pancreatic
2- Oesophagus
area, the lateral part is in relation with the left
3- Aorta colic flexure, the medial with the small
4- Heart intestine. The areas in contact with the
stomach and spleen are covered by the
5- Trachea
peritoneum of the omental bursa, while that
in relation to the small intestine is covered by
Answer & Comments the peritoneum of the general cavity; behind
Answer: 1- Thymus the latter are some branches of the left colic
vessels. The suprarenal, pancreatic, and colic
The anterior mediastinum is bordered areas are devoid of peritoneum.
anteriorly by the sternum and posteriorly by
the great vessels. It contains the thymus,
[ Q: 1569 ] PasTest 2009 - Basic
lymph nodes, fat, and vessels. Disorders of the
Science
anterior mediastinum are generally thymic,
thyroid (substernal goitre), teratoma (and What would be consistent with femoral nerve
other germ cell tumors), and lymphomas damage in a patient with pelvis trauma?
(Hodgkin's disease, non-Hodgkin's lymphoma). 1- Preserved knee reflex
2- Loss of sensation over the anterior-lateral
[ Q: 1568 ] PasTest 2009 - Basic
aspect of the thigh
Science
3- Loss of power in the biceps femoris muscle
Which organ lies anterior in direct contact
4- Loss of power in the peroneus muscle
with the left kidney without separation by
visceral peritoneum? 5- Reduced power on adduction
1- Spleen
Answer & Comments
2- Left suprarenal
3- Tail of the pancreas Answer: 2- Loss of sensation over the anterior-
lateral aspect of the thigh
4- Left psoas muscle
The femoral nerve may be damaged from
5- Splenic flexure
fractures of the pelvis or femur, or
dislocations of the hip, and hip or hernia
Answer & Comments surgery. It can also be involved in psoas
Answer: 3- Tail of the pancreas abscesses, thigh wounds and frequently in
Two of commonest causes of hypercalcaemia over the medial aspect of the elbow which
in the western world are primary worsens on resisted wrist flexion and forearm
hyperparathyroidism and malignancy. In pronation. In severe cases there is weakness
primary hyperparathyroidism there is excess of muscles of the hand and loss of sensation in
production of parathyroid hormone (PTH); the ulnar nerve distribution. Medical
although usually from a benign adenoma, this intervention with appropriate physiotherapy
sometimes results from hyperplasia of the is the mainstay of treatment, rarely surgical
parathyroid glands and, in rare cases, a release is indicated, but is usually successful.
carcinoma. Thyrotoxicosis can cause
hypercalcaemia as well as osteoporosis. The [ Q: 1574 ] PasTest 2009 - Basic
milk-alkali syndrome can occur in patients Science
who suffer from dyspepsia and drink milk and
alkali-containing antacids, which may reduce A 64-year-old man is admitted with a severe
the renal excretion of calcium. Around one- haematemesis. Upper GI endoscopy identifies
fifth of those with sarcoid have increased a posterior gastric ulcer.
calcium levels. Various mechanisms cause Bleeding is most likely to having occurred from
raised hypercalcaemia of malignancy. which main vessel?
1- Splenic artery
[ Q: 1573 ] PasTest 2009 - Basic
2- Left gastroepiploic artery
Science
3- Inferior pancreaticoduodenal artery
A 22-year-old golfer presents for review. He
has noticed a decrease in his grip strength 4- Oesophageal branch of the left gastric
over the past few weeks and pain over his artery
elbow. On examination he has pain over his 5- Gastroduodenal branch of the right gastric
elbow which is worse on wrist flexion and artery
resisted forearm pronation. There is weakness
of the adductor policis muscle and loss of Answer & Comments
pinch grip and loss of sensation over the
lateral fingers of the hand. Answer: 1- Splenic artery
Which of the following is the most likely cause A posterior gastric ulcer may adhere to, and
for his injury? ulcerate, the splenic artery as this runs along
1- Lateral epicondylitis the upper border of the pancreas, resulting in
a drenching haemorrhage. A lesser curve
2- Radial neuropathy gastric ulcer may implicate the left gastric
3- Median neuropathy artery - the gastroepiploic vessels lie along the
4- Medial epicondylitis greater curve of the stomach. A posterior
duodenal ulcer may erode the gastroduodenal
5- Brachial plexus injury branch of the right gastric artery - ‘the ulcer of
duodenal haemorrhage' - the inferior
Answer & Comments pancreaticoduodenal artery supplies the lower
part of the second part of the duodenum, well
Answer: 4- Medial epicondylitis
clear of the site of ulceration. Oesophageal
Medial epicondylitis occurs with increased varices commonly extend into the upper
frequency in golfers and patients who have stomach and are, of course, venous in origin.
excessive use of the elbow joint such as
basketball players. Symptoms include pain
of nerve roots below the end of the spinal Paralysis of the deltoid muscle causes
cord. MRI or CT scanning of the lower spine is weakness of shoulder abduction, particularly
the investigation of choice, with initial pain after 30 degrees of abduction. The
relief the cornerstone of management. Where supraspinatus initiates abduction of the
a cause for compression is identified, such as shoulder in the first 30 degrees of movement.
intervertebral disc herniation, neurosurgical Therefore, this part of shoulder abduction
intervention is of value. may be spared (especially in athletes
presenting sports injury to the axillary nerve,
[ Q: 1578 ] PasTest 2009 - Basic for example, who may have hypertrophied
Science supraspinati). However even if the
supraspinatus is fully functional, it is too weak
A 78-year-old man had poliomyelitis as a child, a muscle to be able to abduct the whole
which left him with total paralysis of the left weight of the arm, especially in an elderly man
deltoid muscle. with a longstanding neuromuscular deficit.
Which feature is most likely to be present on The deltoid, in addition to being the powerful
clinical examination? abductor of the elbow, also assists in flexion
and medial rotation (and extension and lateral
1- Anaesthesia over the ‘epaulette’ region of
rotation) of the shoulder by means of its
the left shoulder
anterior and posterior fibres, respectively.
2- The acromion process of the scapula forms Weakness of these movements compared to
the most lateral bony landmark of the left the normal side can be detected on careful
shoulder examination.
3- Drooping of the left shoulder compared to
the right side [ Q: 1579 ] PasTest 2009 - Basic
4- Detectable weakness in drawing the arm Science
forward and internally rotating the An 84-year-old man had his left sciatic nerve
shoulder when this is compared with the completely transected just inferior to the
right side buttock crease by a piece of shrapnel during
5- Abduction of the shoulder to 60o is likely to the D-day landings in 1944.
be preserved due to action of the intact Which sign is likely to be present on current
supraspinatus muscle on the left side neurological examination?
1- Complete anaesthesia below the knee
Answer & Comments
2- Spastic paralysis of the lower limb, with
Answer: 4- Detectable weakness in drawing increased ankle jerk
the arm forward and internally rotating the
3- Plantar flexed and everted foot
shoulder when this is compared with the right
side 4- Paralysed quadriceps femoris
sensation along the medial side of the anterior home. On examination there is weakness of
aspect of the leg down to the base of the the foot and ankle dorsiflexors and she has a
hallux. Peripheral nerve injuries result in so-called steppage gate. She has sensory loss
flaccid paralysis, with loss of stretch reflexes over the lateral portion of the leg extending
of the paralysed muscles. The foot is plantar onto the dorsum of the foot.
flexed due to gravity, (foot drop). However, it Which of the following is the most likely
is not everted (eversion is produced by the location of neurological injury?
peroneus longus and brevis, which are
paralysed), nor is it inverted, a function of the 1- Femoral nerve injury
paralysed long flexors of the foot. The 2- Common peroneal nerve injury
quadriceps is innervated by the intact femoral 3- Tibial nerve injury
nerve, and hip abduction, effected by the
gluteus medius and minimus, supplied by the 4- L3 nerve lesion
intact superior gluteal nerve, is unaffected. 5- L4 nerve lesion
Which of the following biochemical changes Answer: 5- It may be used in the treatment of
would you most expect to occur first? blepharospasm
1- Fall in serum free thyroxine Clostridium botulinum is a Gram-positive,
2- Fall in serum thyroxine-binding globulin spore-forming, obligate anaerobe. The bacillus
has seven serotypes, A to G. They have a wide
3- Fall in serum free triiodothyronine
range of therapeutic usage, from glabellar
4- Fall in serum total triiodothyronine lines, blepharospasm, spasticity, anismus, anal
5- Increase in serum TSH fissure to dystonia. However, myasthenia
gravis would be expected to worsen with such
treatment.
Answer & Comments
Answer: 5- Increase in serum TSH [ Q: 1585 ] PasTest 2009 - Basic
Hypothyroidism develops gradually, often Science
over many months or even years. In the early A 25-year-old patient presents with anaemia
stages, free thyroxine concentrations are and jaundice. A blood film shows
maintained in the normal range by the
[ Q: 1588 ] PasTest 2009 - Basic You are asked to see a 19-year-old man who
Science has had recurrent pneumothoraces. He is tall
with pes planus and has an increased arm
Which of the following statements is true
span to height ratio and upper segment to
about the matrix metalloproteinases, which
lower segment body ratios.
play a major role in pathological processes,
including rheumatoid arthritis, periodontitis, Which other feature would be most helpful in
vascular disease as well as tumour invasion making a diagnosis of Marfan's syndrome?
and metastasis? 1- Joint hypermobility
1- All are controlled by specific tissue 2- High arched palate
stimulators of the metalloproteinases
3- Mid-systolic click
2- All contain an iron atom
4- Arachnodactyly
3- Each is involved in the synthesis of at least
5- Early diastolic murmur
one component of the extracellular matrix,
basement membrane proteins and
bioactive mediators Answer & Comments
currently made on clinical criteria. These cover pathogenesis of RNA viruses, such as HIV,
the skeletal, eye, cardiac, respiratory and skin allowing their genetic code to be integrated
systems and are divided into major and minor into the host genome as DNA. It is also very
criteria. Although arachnodactyly, high arched important in molecular biology, allowing
palate, mitral valve prolapse and joint segments of DNA to be produced from mRNA.
hypermobility are all features of Marfan's
syndrome, they are relatively non-specific [ Q: 1591 ] PasTest 2009 - Basic
(minor criteria). On the other hand, an early Science
diastolic murmur indicates aortic valve
incompetence, which is likely to be secondary A 27-year-old woman presents with
to aortic root dilatation, a major feature of worsening back pain. As a child, she and her
Marfan's syndrome. This is the most serious sister had several admissions to hospital with
complication of Marfan's syndrome and bone fractures following minor trauma and
affected individuals require annual cardiology she has recently developed bilateral hearing
assessments. loss. Apart from some teeth discoloration and
mild joint laxity, examination was normal. A
Other major skeletal features include chest vertebral X-ray showed partial collapse of L3.
wall deformities (pectus carinatum or pectus
What is the most likely diagnosis?
excavatum, requiring surgery), a reduced
upper segment to lower segment ratio, an 1- Osteogenesis imperfecta
arm span to height ratio > 1.05, a scoliosis of > 2- Early-onset osteoporosis
20 degrees and pes planus. Lens dislocation
3- Ehlers-Danlos syndrome
(usually upwards) is found in 50-80% of
affected individuals. 4- Malignant bone metastases
5- Multiple myeloma
[ Q: 1590 ] PasTest 2009 - Basic
Science Answer & Comments
You are teaching molecular biology to a group Answer: 1- Osteogenesis imperfecta
of medical students, and one asks you about
how reverse transcriptase works. Osteogenesis imperfecta is a dominantly
inherited disorder of collagen, and this woman
How does reverse transcriptase work?
most probably has the type 1 (least severe)
1- It amplifies segments of DNA form. Type 2 is lethal in utero or shortly after
2- It cleaves specific portions of DNA birth and type 3 is the severe form where
affected individuals have hundreds of
3- It is involved in protein synthesis
fractures and usually become wheelchair-
4- It transcribes RNA from DNA bound. Clinical features include blue sclerae
5- It transcribes DNA from RNA (not always present, as in this case), early-
onset deafness and dentinogenesis
imperfecta.
Answer & Comments
Ehlers-Danlos syndrome is another
Answer: 5- It transcribes DNA from RNA
collagenopathy. The classical type is typically
Reverse transcriptase is a DNA polymerase associated with skin laxity and joint
enzyme that is involved in transcribing single hypermobility but not with susceptibility to
stranded portions of RNA into lengths of bone fractures.
double stranded DNA. Reverse transcriptase
has an important role to play in the
The other conditions should be considered tubers, but these would not explain the
and excluded. However, none would explain history of sudden-onset headache.
her previous history of bone fractures, teeth
discoloration or hearing loss. [ Q: 1593 ] PasTest 2009 - Basic
Science
[ Q: 1592 ] PasTest 2009 - Basic
What is the main feature of DNA sequence
Science
polymorphisms that differentiates them from
A 38-year-old woman is sent as an emergency mutations?
to you with an acute-onset headache and 1- They are common in the population
deteriorating conscious level. Her husband
mentions that her brother has recently had a 2- They often cause serious medical conditions
kidney transplant, although he is not sure 3- They usually interfere with normal gene
why. function
What condition may be running in the family? 4- They are not found in certain ethnic groups
1- von Hippel-Lindau disease 5- They are evenly spread throughout all
2- Hereditary haemorrhagic telangiectasia genes
Acute intermittent porphyria (AIP) presents from his mother who must be a carrier (she
with gastrointestinal and neurological may even be homozygous as women in
symptoms. Central abdominal pain and bilious particular are often asymptomatic). In this
vomiting are characteristic. AIP occurs due to way, although it appears to be dominantly
the absence of porphobilinogen (PBG) inherited, it is in fact a recessive condition that
deaminase. This leads to elevated levels of 8- has been inherited in a pseudo-dominant
ALA and PBG. Attacks are intermittent, more manner. Pseudo-dominant inheritance is a
common in women and may be precipitated feature of diseases with a high carrier
by a number of drugs. There are no skin frequency or consanguineous families. Both
manifestations. Both variegate porphyria and mitochondrial and X-linked inheritance do not
hereditary coproporphyria are associated with exhibit male-male transmission and non-
photosensitive blistering skin lesions. paternity does not explain the inheritance
Erythropoietic protoporphyria and porphyria pattern.
cutanea tarda are cutaneous porphyrias. In
the majority of these patients subepidermal [ Q: 1598 ] PasTest 2009 - Basic
bullae and hypertrichosis develop. In Science
erythropoietic protoporphyria, the patient
develops a burning sensation affecting light- You review a 21-year-old man with albinism.
exposed parts of the skin. Urinary PBG and He is from Central/ South America and you
ALA levels are normal in both these believe that he has Hermansky-Pudlak
conditions. syndrome (HPS). You believe that this is due
to a defect in the transport of glycoproteins
from the Golgi body.
[ Q: 1597 ] PasTest 2009 - Basic
Science Which of the following best fits the position of
the Golgi body within the cell?
A 70-year-old Irish man is diagnosed with
haemochromatosis. Subsequently, his son is 1- In the nucleus
also diagnosed with the condition. Both are 2- Adjacent to the endoplasmic reticulum
homozygous for the common Northern 3- Moves within the cytoplasm
European mutation.
4- Sits across the cell membrane
What is the most likely explanation?
5- Just inside the cell membrane
1- Autosomal-dominant inheritance
2- Mitochondrial inheritance Answer & Comments
3- Non-paternity
Answer: 2- Adjacent to the endoplasmic
4- Pseudo-dominant inheritance reticulum
5- X-linked inheritance The Golgi body is one of the largest cellular
organelles and is found adjacent to the
Answer & Comments nucleus. It was first described by Camillo Golgi
in 1897 and is responsible for modification of
Answer: 4- Pseudo-dominant inheritance
proteins and lipids produced by other cellular
Haemochromatosis is an autosomal-recessive organelles. HPS results from a mutation in the
condition. However, the carrier frequency in AP3 gene responsible for trafficking specific
northern Europeans is high (1 in 10); in this glycoproteins from the Golgi body. It is
case the son must have inherited one mutant inherited in autosomal recessive fashion.
allele from his father and one mutant allele
[ Q: 1599 ] PasTest 2009 - Basic oxygen and carbon dioxide can also diffuse
Science through the bilayer, passing easily through the
temporary small spaces between the tails of
Phase 0 of the cardiac action potential relates
the phospholipids.
to a:
1- Rapid efflux of calcium (Osmosis is the diffusion of water (not
particles) across a membrane, in which water
2- Rapid influx of calcium diffuses into a solution having a greater solute
3- Rapid influx of potassium concentration).
4- Rapid influx of sodium
[ Q: 1601 ] PasTest 2009 - Basic
5- None of the above
Science
worse in males for X-linked trinucleotide may be given for disseminated herpes
repeat disorders such as Fragile X but is not infection or in immunocompromised
true for autosomal trinucleotide repeat individuals.
disorders. Before the molecular mechanism
was understood, ascertainment bias was a [ Q: 1608 ] PasTest 2009 - Basic
seriously considered alternative explanation Science
and it can still be a contributing factor, eg a
person is more likely to be diagnosed earlier You are asked to help identify whether the
with Huntington's disease if there is a known child of a woman with a neuromuscular
family history. disorder is affected by the condition. A
specific DNA sequence associated with the
mutation which causes the disorder has been
[ Q: 1607 ] PasTest 2009 - Basic
identified.
Science
Which of the following methods is a technique
A 74-year-old man receives an acyclovir to identify a particular DNA sequence?
shingles treatment pack from his GP. He is a
retired chemist and is interested in asking you 1- Northern blot
about how acyclovir works. 2- Eastern blot
Which of the following best describes the step 3- Southern blot
required for acyclovir activation? 4- Western blot
1- Conversion to diphosphate form by viral 5- South-Western blot
thymidine kinase
2- Conversion to monophosphate form by Answer & Comments
viral thymidine kinase
Answer: 3- Southern blot
3- Metabolism by intracellular phosphatases
4- Reverse transcriptase activation Southern blotting was a technique developed
by Edwin Southern, a British biologist. It
5- Viral protease activity combines agarose gel electrophoresis for size
separation of DNA with use of a filter
Answer & Comments membrane for probe hybridisation. Northern
blotting studies sequences of RNA, whereas
Answer: 2- Conversion to monophosphate
Western blotting is used to look for specific
form by viral thymidine kinase
proteins. Southwestern blotting is used to
Acyclovir is a guanine analogue pro-drug used study DNA binding proteins.
primarily for the treatment of herpes zoster or
simplex infection. Conversion from a partial [ Q: 1609 ] PasTest 2009 - Basic
nucleoside structure to a monophosphate Science
form by viral thymidine kinase allows it to
incorporate into viral DNA resulting in chain You are asked to review a 17-year-old who
termination. Viral DNA polymerase is then suffers from leprachaunism. You understand
unable to function and replication of the that this occurs due to a mutation associated
Herpes virus is terminated. Peak plasma with the insulin receptor.
concentration is reached 1-2hrs after oral Where is the insulin receptor located?
administration of acyclovir and the drug is
1- Cell membrane
fairly rapidly eliminated, this necessitates
multiple daily dosing regimens. IV acyclovir 2- Nucleus/nuclear membrane
A patient has been investigated for loose Cystic fibrosis (CF) is an autosomal recessive
joints. He also complains of reduced vision. On disorder that is caused by mutations in the
examination he has very thin extremities and cystic fibrosis transmembrane conductance
arachnodactyly. regulator (CFTR) gene on 7q31.2. Affected
individuals have mutations in both alleles of
What is the most likely diagnosis?
this gene. The siblings of an affected individual
1- Marfan's syndrome have:
2- Tay-Sachs disease
a 1 in 4 (25%) chance of being affected with
3- Fabry disease this condition
4- Gaucher disease a 1 in 2 (50%) chance of being CF carriers
5- Niemann-Pick disease
a 1 in 4 (25%) chance of being neither affected
nor a carrier.
Answer & Comments
The proband in this question is a healthy 20-
Answer: 1- Marfan's syndrome year-old man and he is therefore unaffected
Marfan's syndrome is characterised by a triad with CF. When affected siblings are removed
of features: from the above equation then the unaffected
siblings have a 2 in 3 (67%) chance of being CF
long, thin extremities frequently associated carriers and 1 in 3 (33%) chance of not being
with other skeletal changes, such as loose carriers.
joints and arachnodactyly
aortic aneurysms that typically begin at the A 40-year-old man develops jerky movements
base of the aorta. affecting various parts of his body. His father
A 20-year-old woman, who was prescribed the What is the most likely diagnosis?
oral contraceptive pill a week earlier, develops 1- Infectious mononucleosis
central abdominal pain, vomiting and
2- Diphtheria
weakness in both lower limbs. Blood tests
show an elevated white cell count. 3- Rubella
considered. Of course, the situation would 6 of the b-globin chain is the genetic
have to be handled very delicately. Non- abnormality encountered in which one of the
paternity rates are notoriously difficult to following types of congenital haemolytic
measure. Many studies have found very anaemia?
variable results, from 1-30% or even higher in 1- Sickle cell anaemia
some age groups.
2- β-Thalassaemia
Phaeochromocytomas are not associated with massive expansion in the triplet repeat that is
hereditary haemorrhagic telangiectasia (HHT) inherited from the affected mother.
or neurofibromatosis type 2 (NF2). However,
The other listed conditions are not associated
brain tumours (vestibular schwannomas,
with ptosis, although cardiac problems
gliomas, meningiomas) are a feature of NF2
(typically cardiomyopathy) can occur in
and cerebral arteriovenous malformations are
Duchenne's muscular dystrophy and its milder
sometimes seen in patients with HHT.
variant, Becker's muscular dystrophy.
Duchenne's muscular dystrophy usually
[ Q: 1644 ] PasTest 2009 - Basic results in death by the third decade.
Science Facioscapulohumeral dystrophy is
A 40-year-old man is referred to you with characterised by a pattern of muscle
breathlessness and bradycardia. During the weakness involving the face, scapula, upper
consultation you notice that he has a bilateral arm, lower leg and hip girdle. Spinal muscular
ptosis. He is estranged from his family and atrophy is an autosomal-recessive condition
never knew his father, but he does know that usually affecting children, but there is a milder
his sister has a muscle problem and lost a adult version (type IV) which causes a
child in infancy. proximal myopathy.
Imprinting is a mechanism that results in 4- They are proteins that attract phagocytes
differential gene expression according to 5- They perforate invading bacteria
parental origin. It is responsible for a number
of genetic syndromes, the best known of
Answer & Comments
which are the Angelman's and Prader-Willi
syndromes. Angelman's syndrome causes Answer: 4- They are proteins that attract
ataxia and severe learning disability and is phagocytes
caused by absent maternal expression of the
15q11-13 chromosome region. Conversely, Kinins are proteins that attract phagocytes,
the Prader-Willi syndrome (mild/moderate promote vasodilatation and increase the
learning disability, hyperphagia and obesity) is permeability of blood vessels. Complement
caused by absent paternal expression of the perforates invading bacteria, dilates blood
15q11-13 chromosome region. Other vessels, stimulates histamine release and
syndromes caused by imprinting attracts neutrophils.
abnormalities include the Beckwith-
Wiedermann syndrome (macroglossia, [ Q: 1647 ] PasTest 2009 - Basic
abdominal wall defects, hypoglycaemia, Science
visceromegaly) and the Russell-Silver
A patient in the intensive care unit following
syndrome (small stature). The exact
liver transplant surgery has a metabolic
mechanism of imprinting is not well
alkalosis.
understood but is thought to involve
differential gene methylation. Which of the following biochemical
abnormalities is most specifically indicative of
Both Angelman's and Prader-Willi syndromes this?
occur with equal frequency in both sexes.
1- Acidic urine
Anticipation is a completely different genetic
mechanism where trinucleotide repeat 2- High arterial blood pH (low hydrogen-ion
expansions cause increasingly severe concentration)
phenotypes in successive generations. 3- High arterial partial pressure of carbon
Variations in X chromosome inactivation are dioxide p(CO2)
thought to partly explain the mild phenotypes
4- High plasma bicarbonate concentration
sometimes seen in female carriers of X-linked
disorders. While modifier genes may well 5- Hypochloraemia
explain phenotypic variation between
individuals with the same syndrome, they do Answer & Comments
not explain the differences between the
Angelman's and Prader-Willi syndromes. Answer: 4- High plasma bicarbonate
concentration
Answer & Comments The half-life is defined as the time taken for
plasma levels of a particular chemical entity to
Answer: 4- Laxative abuse
fall by 50%. These results indicate that levels
This patient has a hypokalaemic, fall by 75% every 2 h. Hence the half-life is
hyperchloraemic metabolic acidosis. The most calculated at 1 h. It is worth noting that a
likely cause is gastrointestinal loss of halving of plasma levels may not indicate a
bicarbonate and potassium due to chronic reduction of 50% in activity of the agent, as a
severe laxative abuse. Whilst mild laxative number of agents are metabolized to
abuse results in alkalosis, more severe loss of products, which are also therapeutically or
bicarbonate eventually leads to acidosis. Her metabolically active.
markedly low BMI raises the suggestion of an
eating disorder. Although diuretic abuse [ Q: 1652 ] PasTest 2009 - Basic
would lead to marked potassium loss, the Science
appearance of alkalosis is more likely.
Diagnosis would be supported by urine testing You are asked to see a patient who had a
for metabolites of commonly used laxative chest drain removed 4 days ago. There
agents. appears to be some infection.
What are the stages in the cell biology of
[ Q: 1651 ] PasTest 2009 - Basic normal wound healing?
Science 1- Demolition is the first phase
You are trialing a new antihypertensive agent, 2- Maturation and remodelling can continue
'Wonderone'. As part of the drug- for up to a year
development programme you must assess the 3- Acute inflammation usually lasts for 6-12
half-life of Wonderone. Some results are sent hours
to you for calculation of the half-life:
4- Epithelial cell proliferation is the hallmark
15 mins after iv injection - Wonderone level of the demolition phase
150
5- Collagen deposition is the key process
2 h 15 mins after iv injection - Wonderone during demolition
level 37.5
4 h 15 mins after iv injection - Wonderone Answer & Comments
level 9.4.
Answer: 2- Maturation and remodelling can
Which of the following stems fits best with the continue for up to a year
half-life of this agent?
The first phase in healing by first intention is
1- 90 min
the phase of acute inflammation that lasts up
2- 15 min to 3 days, if uncomplicated. The initiating
3- 1 h factor appears to originate from platelets
activated by mature collagen exposed in the
4- 2 h
wound. Platelets first aggregate then release a
5- 3 h variety of active agents including lysosomal
enzymes, ATP, serotonin and wound
Answer & Comments cytokines. A fibrin clot develops, which
completes haemostasis and provides strength
Answer: 3- 1 h
and support to the wound. The surface dries
to form a scab. Platelets and macrophage
Epithelial cells at the edge of the wound start Macrophages Granular leucocyte Phagocytosis
to proliferate after 24 h and this phase can Both
last for up to 3 weeks. Monocytes Agranular leucocyte Phagocytosis
Finally, the phase of maturation and Mobile
remodelling lasts for up to 12 months, during NK cells Lymphocytes Phagocytosis Mobile
which time the tensile strength of the wound
increases and the random collagen is replaced B cells Lymphocytes Degranulation Mobile
by a more stable form orientated along lines
T cells Lymphocytes Cytokine production
of stress. Mobile
[ Q: 1653 ] PasTest 2009 - Basic *Mobile white blood cells circulate around the
Science body in the bloodstream; static white blood
cells circulate in a specific area, such as the
Concerning the respiratory cell biology of an lungs. NK, natural killer.
asthmatic individual, which of the following is
true?
[ Q: 1654 ] PasTest 2009 - Basic
1- Monocytes are granular and static Science
2- Eosinophils are agranular and mobile A 45-year-old man has severe pulmonary
3- Basophils are granular and their mode of emphysema. A diagnosis of α1-antitrypsin
action is phagocytosis deficiency is being considered.
4- B cells are mobile and their mode of action What is the genotype most typically
is phagocytosis associated with this condition?
5- T cells are mobile and contribution to 1- PiMM
asthma is via cytokine production 2- PiMZ
3- PiSS
4- PiSZ
5- PiZZ
comprises chemotaxis, ingestion within a have ‘cherry-red' spots in their eyes. The
phagosome, intracellular enzymatic incidence of Tay-Sachs is particularly high
degradation and exocytosis. among people of Eastern European and
Askhenazi Jewish descent. Patients and
[ Q: 1660 ] PasTest 2009 - Basic carriers of Tay-Sachs disease can be identified
Science by a simple blood test that measures beta-
hexosaminidase A activity. Both parents must
The accumulation of gangliosidic GM2 in the carry the mutated gene in order to have an
central nervous system of individuals with Tay- affected child. In these instances, there is a
Sachs disease is attributed to: 25% chance with each pregnancy that the
1- Decreased lysosomal hydrolysis child will be affected with Tay-Sachs disease.
Prenatal diagnosis is available if desired.
2- Decreased Golgi stimulation
3- Increased permeability of the blood-brain
[ Q: 1661 ] PasTest 2009 - Basic
barrier
Science
4- Increased receptor-mediated endocytosis
Which of the following statements regarding
5- Increased synthesis in the endoplasmic the eukaryotic cell cycle is correct?
reticulum
1- M phase signifies meiosis
(CK) activity is reported as being 400 U/l. His What is the most likely cause of the abnormal
CK 24 h after admission had been 620 U/l and findings?
had fallen to 415 U/l after a further 48 h. 1- Familial combined hyperlipidaemia
Which of the following is the most likely cause 2- Hepatic steatosis secondary to alcohol
of the high CK when he was reviewed in
outpatients? 3- Non-alcoholic steatohepatitis
The lipid deposits in the palmar creases, diuresis. ‘Normal saline' is the fluid of choice.
probable tuberous xanthomas on the elbows Bisphosphonates are effective, but must be
and elevations of cholesterol and triglyceride given intravenously to have a rapid effect. A
to approximately equal molar concentrations diuretic may be helpful once the patient has
are typical of remnant hyperlipidaemia been rehydrated, but this should be a loop
(familial dysbetalipoproteinaemia, broad beta diuretic, which increase calcium excretion, not
disease). Bile-acid sequestrants reduce a thiazide, as these reduce it. Corticosteroids
cholesterol but may increase triglyceride may occasionally be helpful but are not first-
concentrations. Statins are principally line treatment. Sodium phosphate infusion is
cholesterol-lowering agents. Nicotinic acid dangerous: it lowers the calcium
lowers cholesterol and triglycerides but is concentration rapidly, but risks causing
poorly tolerated, although derivatives may be metastatic calcification.
more acceptable to patients. Omega-3 fish oils
are relatively weak lipid-lowering agents. [ Q: 1676 ] PasTest 2009 - Basic
Fibrates are usually effective treatment for Science
this condition. Genetic predisposition to this
condition is approximately 100 times more Which of the following fluids would be the
prevalent than the condition itself. This most appropriate to replace the fluid being
dyslipidaemia is frequently expressed clinically lost in a patient with a paralytic ileus draining
because of an additional factor, for example 2 litres of fluid a day through a nasogastric
impaired glucose tolerance or frank diabetes, tube?
and such conditions should be sought and, if 1- Compound sodium lactate (Hartmann's
present, be treated appropriately. solution)
2- 5% dextrose
[ Q: 1675 ] PasTest 2009 - Basic
3- 10% dextrose
Science
4- 0.18% sodium chloride with 4% dextrose
A 56-year-old woman with known metastatic
(‘dextrose saline')
breast cancer presents to A&E with a calcium
concentration of 3.22 mmol/l. 5- 0.9% sodium chloride (‘normal saline')
[ Q: 1680 ] PasTest 2009 - Basic is suffering from multiple sclerosis and are
Science considering prescribing beta-interferon for
her.
A 49-year-old woman presents to her
gynaecologist with CIN-3 changes. She has Which of the following best describes the
been reading on the Internet about how the action of beta-interferon?
human papillomavirus interferes with the 1- It increases MHC class-II expression on
programmed death of defective cells and may antigen-presenting cells
predispose to cancers.
2- It leads to increased MHC class-I expression
Which of the following protein names best
3- It is used in the treatment of hepatitis B
identifies the DNA-binding protein with which
the papillomavirus interferes? 4- It may be used as an adjunct in atypical
mycobacterial disease
1- p52
5- It activates macrophage and neutrophil
2- p51
intracellular killing
3- p53
4- p54 Answer & Comments
5- p55 Answer: 2- It leads to increased MHC class-I
expression
Answer & Comments
Alpha- and beta-interferon are produced
Answer: 3- p53 mainly in response to viral infection. They bind
the same cellular receptor and protect
p53 is a DNA-binding protein which induces
uninfected cells by inducing the upregulation
the expression of a number of genes that are
of molecules that inhibit the viral production
involved in programmed cell death. Clearly,
of RNA and DNA. They also induce MHC class-I
p53 gene mutations or products that interfere
expression leading to enhanced lysis of cells
with p53 protein functioning are implicated in
that are already infected with virus. In
carcinogenesis. Initial p53 gene expression
addition, they have antiproliferative actions.
causes genes involved in DNA repair to be
expressed. If DNA repair is too slow or gene Alpha-interferon is used in the treatment of
repair cannot be effected, then other genes chronic hepatitis B and C infections as well as
involved in apoptosis take over, and some forms of leukaemia. Beta-interferon is
programmed cell death ensues. used in the treatment of multiple sclerosis.
Gamma-interferon induces MHC class-II
Both the adenovirus E1B and papillomavirus
expression on antigen-presenting cells via a
E6 gene products bind p53 and interfere with
different receptor to alpha- and beta-
its functioning. Another protein involved in
interferon.
the cell cycle and triggering of apoptosis is the
RB protein, the functioning of which is
interfered with by a number of viruses [ Q: 1682 ] PasTest 2009 - Basic
including papillomavirus, adenovirus and the Science
SV40 virus. A 68-year-old man is admitted to hospital for
elective femoral angioplasty. On examination,
[ Q: 1681 ] PasTest 2009 - Basic he is found to have widespread
Science lymphadenopathy. Blood is taken for 'group
and save'. His cells are not agglutinated by
You are reviewing a 45-year-old woman who
either anti-A or anti-B; his serum does not
[ Q: 1685 ] PasTest 2009 - Basic 4- Severe vomiting 2-4 h after food ingestion
Science 5- Severe vomiting 6-12 h after food ingestion
therapy with intravenous vancomycin and which follows days or months later, includes
teicoplanin. Other antibiotics to which there neurological symptoms, radiculopathies,
may be sensitivity include some macrolides, cardiac problems and arthritis. Isolation from
trimethoprim, sodium fusidate and blood cultures is rare, as is isolation from skin
aminoglycosides. Linezolid and combination lesions or (CSF). Clinical history with serology
therapy with quinupristin and dalfopristin are results are usually used to determine the
other possible options but these should be diagnosis. Early stage disease may be treated
reserved for patients with organisms that are with amoxicillin or doxycycline; later disease is
resistant to other combinations or for those usually treated with 2-4 weeks of iv
with tolerability issues to other combinations. benzylpenicillin or ceftriaxone.
4- Mycoplasma
Answer & Comments
5- Chlamydia pneumoniae
Answer: 4- Change to ampicillin + gentamicin
and treat for 10-14 days total
Answer & Comments
The recommended regime for meningitis
Answer: 3- Borrelia burgdorferi
caused by Listeria spp. is amoxicillin/ampicillin
The biggest clues here are the history of skin for 10-14 days. Listeria monocytogenes is an
rash and walking trip to the USA. Lyme environmental organism that is widely present
disease, a zoonosis affecting deer and other in soil and decaying matter and produces
wild animals, is caused by B. burgdorferi, a infection in both animals and man. Most
spirochaete. Infection is transmitted from commonly implicated foods that may cause
animal to man via a tick vector. The first stage infection include raw vegetables, soft cheeses
of the disease, occurring 7-10 days after and pأ¢t©أs. The infection may cause serious
infection is characterised by erythaema problems in pregnant women, leading to
chronicum migrans. About 300 cases per year premature labour, septic abortion or stillbirth.
are reported in the UK. Initial stages of the Listeria in the elderly may result in
infection are associated with fever, headache, meningoencephalitis as in this case. Standard
myalgia and joint pain. The second stage, treatment is with ampicillin and gentamicin,
Specific tyrosine kinase inhibitors were Granuloma is seen in which of the following
developed after it was recognised that the conditions?
Philadelphia chromosome (translocation 9:22) 1- Syphilis
was associated with bcl-abl tyrosine kinase
2- Typhoid
overexpression. STI571, later known as
imatinib mesylate (Glivec), was therefore 3- Cholera
designed to treat chronic myeloid leukaemia, 4- Amoebiasis
but it was also found to be effective in
5- Shigellosis
gastrointestinal stromal tumours (GIST) -
previously known as leiomyosarcoma. Trials
also suggest that glivec may of benefit in some Answer & Comments
patients with CLL who are Philadelphia Answer: 1- Syphilis
chromosome positive.
A granuloma is a collection of macrophages:
giant cells as a nidus of chronic inflammation.
[ Q: 1698 ] PasTest 2009 - Basic
The centre may necrotise to form caseation,
Science
classically in tuberculosis. There is a long list of
Which of the following antineoplastic agents is infective and immunological conditions where
correctly paired with the site of action at the a granulomatous response may be seen,
cellular level? including tertiary syphilis, sarcoidosis and
1- Vinca alkaloids and abnormal microtubule Crohn's and Wegener's granulomatosis.
disassembly
[ Q: 1700 ] PasTest 2009 - Basic
2- Taxanes and abnormal microtubule
Science
assembly
3- Irinotecan and topoisomerase inhibition Which of the following statements is true
regarding the cellular and molecular
4- Imatinib mesylate and competitive
mechanisms that control apoptosis?
inhibition of nucleotide synthesis
1- The extrinsic pathway is initiated at the
5- Doxorubicin and competitive inhibition of
mitochondrial level
nucleotide synthesis
2- The intrinsic pathway is triggered at death
receptors on the cell surface
Answer & Comments
3- Abnormalities of caspase control are
Answer: 3- Irinotecan and topoisomerase associated with a range of lymphomas and
inhibition carcinomas
Vinca alkaloids inhibit the assembly of 4- Caspase 8 is an example of an executioner
microtubules, while taxanes inhibit the normal protein
5- Death domains attract extracellular adapter lymphoma, and surviving with a number of
proteins carcinomas. Mutations of the p53 system are
seen in a variety of colorectal and other
Answer & Comments neoplasms. Finally, upregulation of other
mediating proteins, such as FLIP, can be seen
Answer: 3- Abnormalities of caspase control in certain tumours.
are associated with a range of lymphomas and
carcinomas
[ Q: 1701 ] PasTest 2009 - Basic
Apoptosis operates within two different Science
signalling pathways. The extrinsic pathway is
You are investigating periodic episodes of
initiated by ligand binding, the intrinsic
unexplained hypoglycaemia in a 23-year-old
pathway at the mitochondrial level. All death
nurse and request a C-peptide assay.
receptors (DR) are members of the tumour-
necrosis (TNF) superfamily and comprise a Which of the following statements is correct?
subfamily characterised by an intracellular 1- The level of C-peptide will be reduced in
domain, ie the death domain. Examples of insulinoma
known death receptors are CD95 (APO-1/Fas),
2- C-peptide cleavage is an example of post-
TRAIL (TNF- related, apoptosis-induced
transcriptional modification
ligand)-R1, TRAIL-R2, TNF-R1, DR3 and DR6.
3- C-peptide is attached to only the alpha
Death domains attract intracellular adaptor chain
proteins (eg CD95/FADD (MORT-1)), which in
turn attract procaspase 8. The latter is 4- The level of C-peptide will be unchanged in
factitious hypoglycaemia
modified into a complex of two small and two
large heterodimers, constituting active 5- C-peptide cleavage in an example of post-
caspase 8. This activates downstream translational modification
caspases, eg caspase 3 (a typical executioner
caspase) and results in the host cell Answer & Comments
undergoing characteristic apoptotic changes,
such as chromatin condensation and cell Answer: 5- C-peptide cleavage in an example
fragmentation (blebbing). of post-translational modification
Adherent junctions are continuous on the DNA photolyase is a bacterial enzyme involved
basal side of cells. They contain cadherins and in photoreactivation repair.
are the major site of attachment of
DNA glycolyase takes part in base excision and
intracellular microfilaments. Intermediate
mismatch repair processes.
filaments are attached to desmosomes, which
are thickened areas of two apposed adjacent
cell membranes. In blistering skin disorders [ Q: 1704 ] PasTest 2009 - Basic
autoantibodies cause damage by attacking Science
proteins such as desmoglein-3 in pemphigus A 50-year-old obese patient has been
vulgaris, and desmoglein-1 in pemphigus complaining of gradually increasing firm
foliaceus. This leads to disruption of the nodules over the extensor surfaces of his
adherent junctions between cells and results fingers, hands and forearms. He also noticed
in blister formation. similar ear lesions.
Claudins are proteins involved in tight Which is the most likely enzyme when
junctions between cells. Tight junctions exist inhibited to cause a reduction of these
at the ends of margins adjacent to epithelial deposits?
cells (eg intestinal and renal cells) and form a
1- Urease
barrier to the movement of solutes and ions
across the epithelium. Connexons are protein 2- Xanthine oxidase
channels formed between two adjacent cells 3- Hypoxanthine phosphoribosyltransferase
to allow the passage of solutes up to a
4- Adenylate kinase
molecular weight of 1000 kDa.
5- Creatine kinase
[ Q: 1703 ] PasTest 2009 - Basic
Science Answer & Comments
Which blood vessels are most sensitive to the of ATP.Riboflavin is absorbed from the upper
vasodilatatory effect of nitrates? gastrointestinal tract, there is no specific
1- Large arteries storage tissue, and it is excreted in the urine
either free or in small amounts of
2- Coronary arteries hydroxylated products.
3- Capillaries
4- Large veins [ Q: 1711 ] PasTest 2009 - Basic
Science
5- Pulmonary arteries
The gene for which of the following disorders
Answer & Comments is correctly paired with the stated
chromosome?
Answer: 4- Large veins
1- Duchenne muscular dystrophy: X
The antianginal and haemodynamic effects chromosome
are mediated predominantly by vasodilatation
2- Haemophilia A: Chromosome 11
of the venous system, leading to a fall in left
ventricular preload and cardiac work. 3- Variegate porphyria: X chromosome
4- Cystic fibrosis: Chromosome 1
[ Q: 1710 ] PasTest 2009 - Basic 5- Hereditary haemochromatosis :
Science Chromosome 7
For what metabolic process is riboflavin
required? Answer & Comments
1- The synthesis of the protein, collagen Answer: 1- Duchenne muscular dystrophy: X
2- The synthesis of glycogen from glucose chromosome
phosphate The gene for variegate porphyria (the PPOX
3- The synthesis of non-essential amino acids gene) has been located to chromosome 1. It
4- Prostaglandin synthesis has an autosomal-dominant pattern of
inheritance. Hereditary haemochromatosis is
5- The hydrogen-transfer chain in the an autosomal-recessive disorder with variable
mitochondria penetrance, and results from a mutation of
the HFE gene located on chromosome 6. This
Answer & Comments encodes a transport protein found in the
duodenum. 77.5% of patients have two copies
Answer: 5- The hydrogen-transfer chain in the
of the C282Y mutation, 4% have one copy,
mitochondria
while 6.5% have one or two copies of a
Riboflavin is a substituted alloxazine ring different mutation, H63D. Cystic fibrosis is
linked to ribotol, an alcohol derived from the caused by a mutation of the CFTR gene, which
pentose sugar ribose. It is light sensitive. is normally found on chromosome 7.
Dietary sources are liver, milk, cheese, eggs, Haemophilia, Duchenne and Becker forms of
some green vegetables, and beer. Other muscular dystrophy are all X-linked recessive
sources are yeast extracts (for example, conditions.
Marmite) and meat extracts (for example,
Bovril).Riboflavin is involved in redox [ Q: 1712 ] PasTest 2009 - Basic
processes involving the hydrogen-transfer Science
chain in the mitochondria and the production
Which one of the following muscles in the